Sei sulla pagina 1di 116

Copyright © 2014 Delhi Academy of Medical Sciences, All Rights Reserved.

1/116
Test Information
Test Name MD/MS-GRAND TEST -166-(2019) Total Questions 300

Test Type Examination Difficulty Level Difficult

Total Marks 1200 Duration 210minutes

Test Question Language:- ENGLISH


(1). Tip of tongue drains lymphatics into:

a. Occipital lymph node

b. Submental lymph node

c. Deep cervical lymph nodes

d. Tonsiliar lymph nodes

Solution. b.

Submental lymph node Reference – Read the text below Sol:


− Apical vessels drain the tip and inferior surface of the tongue into submental lymph nodes after piercing the mylohyoid muscle.
− Their efferents go to the submandibular nodes mainly, some cross the hyoid bone to reach the jugulo-omohyoid nodes.

Correct Answer. b

(2). Neural crest cells give rise to all the dental structures EXCEPT:

a. Odontoblasts

b. Dentine

c. Enamel

d. Tooth pulp

Solution. c. Enamel
• Enamel is formed by ameloblast cells developing in surface ectoderm of oral cavity.
• The dental papillae are formed by a condensation of neural crest mesenchyme that underlie the enamel organs, and give rise to the
odontoblasts (which form dentin) and dental pulp.

Correct Answer. c

(3). Lesser petrosal nerve derives preganglionic fibers from:

a. Tympanic nerve

b. Cochlear nerve

c. Nervusintermedius

d. Chordatympani

Solution. a. Tympanic nerve


• Inferior salivatory nucleus sends preganglionic parasympathetic fibres through tympanic branch of glossopharyngeal nerve, which
forms tympanic plexus in the middle ear cavity, and sends fibres through lesser petrosal nerve to reach the otic ganglion.

Correct Answer. a

(4). Parotid duct opens:

a. Opposite 1st molar

b. Opposite 2nd molar

c. Into the papilla

Copyright © 2014 Delhi Academy of Medical Sciences, All Rights Reserved. 2/116
d. Opposite incisor

Solution. b. Opposite 2nd molar


• Stensen’s parotid duct crosses the masseter, pierces the buccinator muscle, and opens into the vestibule of the oral cavity opposite the
upper second molar tooth.

Correct Answer. b

(5). Nerve supply of the tip of nose is :

a. Infratrochlear

b. Supratrochlear

c. Supraorbital

d. External nasal

Solution. (b) External nasal


Ref: Read the text below
Sol:
The tip of nose is supplied by external nasal a branch of ophthalmic division of trigeminal nerve.

Correct Answer. b

(6). Nerve which loops around submandibular duct?

a. Mandibular nerve

b. Lingual nerve

c. Hypoglossal nerve

d. Recurrent laryngeal nerve

Solution. b. Lingual nerve


• Submandibular duct runs medial to and then superior to the lingual nerve (lingual nerve loop under submandibular duct).

Correct Answer. b

(7). Lobes of submandibular gland are divided by which muscle?

a. Mylohyoid

b. Genioglossus

c. Stylohyoid

d. Styloglossus

Solution. a. Mylohyoid
• Submandibular gland wraps around the posterior border of mylohyoid.
• Thus, the gland has a large part superficial to the muscle and a small part which lies deep to the muscle.

Correct Answer. a

(8). All are true about palatine tonsil EXCEPT:

a. Develop from 2nd pharyngeal pouch

b. Irritation cause referred pain in ear via auricular branch of vagus

c. Mainly supplied by facial artery

d. Situated in the lateral wall of the oropharynx

Copyright © 2014 Delhi Academy of Medical Sciences, All Rights Reserved. 3/116
Solution. b. Irritation cause referred pain in ear via auricular branch of vagus
• Tonsil develops in the region of second pharyngeal pouch, which forms the tonsillar epithelium.
• It is present on each side of the oropharynx in an interval between the palatoglossal and palatopharyngeal folds and mainlysupplied by
facial artery branches.
• It receives branches of the glossopharyngeal nerve and the lesser palatine branch of the maxillary nerve.
• Since the glossopharyngeal nerve also supplies ear region, any pathology of the tonsil and tonsillar fossa may be accompanied by pain
referred to the ear.

Correct Answer. b

(9). Soft palate is supplied by:

a. Cranial accessory

b. Trigeminal

c. Abducent

d. spinal accessory

Solution. a. Cranial accessory


• Most of the palate muscles are supplied by the cranial accessory nerve, fibres carried by the vagal branches (vagus accessory
1111111complex; pharyngeal plexus), except tensor palati (which is supplied by the mandibular nerve, trigeminal).

Correct Answer. a

(10). Lower border of pharynx is the level of:

a. C2

b. C3

c. C4

d. C6

Solution. d. C6
• Pharynx, the upper portion of gut tube, is funnel-shaped fibromuscular tube that extends from the base of the skull to the inferior border
of the cricoid cartilage at the level of C6 vertebra.
• It continues inferiorly as the esophagus.

Correct Answer. d

(11). What is the arterial supply of the thoracic esophagus?

a. Inferior thyroid artery

b. Inferior phrenic artery

c. Bronchial artery

d. Left gastric artery

Solution. c. Bronchial artery


• Thoracic esophagus is supplied by the branches of descending thoracic aorta like bronchial arteries.
• Inferior thyroid artery supply cervical esophagus.
• Inferior phrenic artery and left gastric artery supply the abdominal esophagus.

Correct Answer. c

(12). Which of the following sensation is not perceived by spinal nucleus of trigeminal nerve?

a. Pain

b. Touch

c. Temperature

d. Proprioception

Copyright © 2014 Delhi Academy of Medical Sciences, All Rights Reserved. 4/116
Solution. D
Exp: Proprioceptionis perceived by mesencephalic nucleus of trigeminal nerve.

Correct Answer. d

(13). Artery beneath red mark in the image below.

a. Middle meningeal

b. Middle cerebral

c. Deep temporal

d. None

Solution. A. Middle meningeal


• The pterion is the region where the frontal, parietal, temporal, and sphenoid bone join together.
• Anterior division of middle meningeal artery lies deep to it.
• It is located on the side of the skull, just behind the temple.

Correct Answer. a

(14). Which of the following is the pre trematic nerve of first pharyngeal arch?

a. Mandibular nerve

b. Chorda tympani

c. Maxillary nerve

d. Ophthalmic nerve

Solution. B. Chorda tympani


- The mandibular and maxillary branches of the trigeminal nerve (CN V) innervate the structures derived from the corresponding
processes of the first arch.
- The nerve of the arch itself runs along the cranial side of the arch and is called post-trematic nerve of the arch.
- Each arch also receives a branch from the nerve of the succeeding arch called thepretrematic nerve which runs along the caudal border
of the arch.
- In human embryo, a double innervation is seen only in the first pharyngeal arch.
- The mandibular nerve is the post-trematic nerve of the first arch and chorda tympani (branch of facial nerve) is thepretrematic nerve.
- This double innervation is reflected in the nerve supply of anterior two-thirds of tongue which is derived from the first arch.

Correct Answer. b

Copyright © 2014 Delhi Academy of Medical Sciences, All Rights Reserved. 5/116
(15). A neuroscientist is investigating the development of the nervous system. In his experiments, he microinjects a dye into the embryo of an
animal subject in vivo. After birth, he performs histological studies to determine the destination of the dye. In one animal subject,he
locates the dye in the dorsal horn of the spinal cord. Which of the following locations in the embryo was the most likely site of the
injection?

a. Alar plate

b. Basal plate

c. Neural crest

d. Rostral end of neural tube

Solution. (a) Alar plate


Ref– Read the text below
Sol:
- The spinal cord arises from the caudal end of the neural tube. During development, an alar and a basal plate is formed, separated by a
longitudinal groove called the sulcus limitans.
- The alar plate forms the dorsal (posterior) part of the spinal cord and becomes the sensory or afferent portion of the cord. The basal
plate is the ventral (anterior) part of the cord, and becomes the motor, or efferent, portion of the spinal cord, and therefore would contain
anterior horn cells.
- The neural crest develops into multipolar ganglion cells of autonomic ganglia, pseudounipolar cells of spinal and cranial nerve ganglia,
leptomeningeal cells, Schwann cells, melanocytes, chromaffin cells of the adrenal medulla, and odontoblasts.

Correct Answer. a

(16). In the image given below, which of the following structures does not pass through the encircled foramen?

a. Sensory root of mandibular nerve

b. Maxillary nerve

c. Lesser superficial petrosal nerve

d. Motor root of trigeminal nerve

Solution. (b) Maxillary nerve


Ref– Read the text below
Sol:
The bone depicted in the figure is sphenoid. The encircled part reperesents the foramen ovale. The maxillary nerve passes through
foramen rotundum.
STRUCTURES PASSING THROUGH FORAMEN OVALE –
- Mandibular nerve
- Accessory meningeal artery
- Lesser petrosal nerve
- Emissary vein connecting the cavernous sinus with the pterygoid plexus of veins

Correct Answer. b

(17). The nucleus that lies deep to facial colliculus is –

a. Trigeminal nucleus

b. Abducens nucleus

c. Facial nucleus

d. Hypoglossal nucleus

Copyright © 2014 Delhi Academy of Medical Sciences, All Rights Reserved. 6/116
Solution. (b) Abducens nucleus
Ref– Read the text below
Sol: Abducens nucleus lies deep to facial colliculus.

Correct Answer. b

(18). Damage to the periamygdaloid, piriform and entorhinal cortex is likely to cause which of the following sensory deficit?

a. Visual

b. Odor and taste

c. Auditory

d. No major sensory deficit.

Solution. (b) odor and taste


Ref: Read the text below
Sol:
Information is transmitted from the olfactory bulb by axons of the mitral and tufted relay neurons in the lateral olfactory tract. Fibres
from the mitral cells terminate on apical dendrites of pyramidal cells in five regions of the olfactory cortex- anterior olfactory nucleus,
olfactory tubercle, piriform cortex, amygdala, and entorhinal cortex. Tufted cells project to anterior olfactory nucleus and olfactory
tubercle.
Conscious discrimination of odor depends on the neocortex (orbitofrontal and frontal cortices). Emotive aspects of olfaction derive from
limbic projections (amygdala and hypothalamus).

Correct Answer. b

(19). There are two electrodes placed at a distance of 4.5cm and it takes 1.5ms for the action potential to be propagated along the nerve from
one electrode to the other electrode. The velocity of nerve conduction is –

a. 60 m/s

b. 30 m/s

c. 45 m/s

d. 80 m/s

Solution. (b) 30 m/s


Ref: Read the text below
Sol:
Velocity=Distance travelled/Time taken to travel distance
4.5cm/1.5ms = 3cm/ms = .03m/.001sec = 30m/sec

Correct Answer. b

(20). Effect of lesion in upper part of pons causes which of the following patterns of breathing?

a. It becomes slower and deeper

b. Apneustic breathing

c. Breathing cases

d. Irregular & gasping

Solution. (a) It becomes slower and deeper


Ref: Read the text below
Sol:
Mid pontine section (or lesion in the upper part of pons) causes loss of pneumotaxic center control. This causes:-
i) With vagi intact:- Increased depth and therefore, decreased rate of breathing- slow and deep breathing.
ii) With bilateral vagotomy:- Apneustic breathing (or apneuses), i.e. sustained or prolonged inspiratory spasm which is interrupted by brief
and inefficient expiration.

Correct Answer. a

(21). The hyper polarization phase of the action potential is due to?

a. Opening of voltage-gated Chloride channels

Copyright © 2014 Delhi Academy of Medical Sciences, All Rights Reserved. 7/116
b. Slow closure of voltage-gated K+ channels
+
c. Slow closure of Na leak channels
-
d. Closure of Cl channels

Solution. (b) Slow closure of voltage-gated K+ channels


Ref: Read the text below
Sol:
- During repolarization phase K+ channels close, but quit slowly. Therefore, more K+
ions leak out, causing a brief period in which the voltage falls 1- 2 mV below -70mV, i.e.
hyperplorization.

Correct Answer. b

(22). Which of the following terms is related to the pathway shown below?

a. Myotactic reflex

b. Inverse stretch reflex

c. Reciprocal innervation

d. Deep tendon jerk

Solution. (b) Inverse stretch reflex


Ref: Read the text below
Sol :
- The pathway shown is inverse stretch reflex also known as autogenic inhibition. The receptor is golgi tendon organ
- When the tension due to the stretch of a muscle exceeds a limit, contraction suddenly ceases and muscle relaxes
- Innervation is by the Ib group of myelinated, rapidly conducting sensory nerve fibres

Correct Answer. b

(23). Purely depolarizing cells are

a. Amacrine cells

b. Rods & cones

c. Bipolar cells

d. Horizontal cells

Copyright © 2014 Delhi Academy of Medical Sciences, All Rights Reserved. 8/116
Solution. (a) Amacrine cells
Ref: Read the text below
Sol:
- The responses of the rods, cones and horizontal cells are hyperpolarizing.
- Bipolar cells may show either hyperpolarization or depolarization.
- Amacrine cells connect ganglion cells to one another in the inner plexiform layer vai processes of varying lengths and patterns.
- At least 29 different types of amacrine cells have been described on the basis of their connections.

Correct Answer. a

(24). Tonic neck reflex is integrated in which of the following?

a. Cerebral cortex

b. Midbrain

c. Medulla

d. Spinal cord

Solution. (c) Medulla


Ref: Read the text below
Sol:

Correct Answer. c

(25). Adrenal insufficiency in not associated with

a. Hyponatremia

b. Hyperkalemia

c. Hypotension

d. Metabolic alkalosis

Solution. (d) Metabolic alkalosis


Ref: Read the text below
Sol:
Features of adrenal insufficiency (Addison’s disease) are:-
I. Hypoglycaemia
II. Hyperpigmentation
III. Hyperkalemia
IV. Hyponatremia
V. Hypotension
VI. Metabolic acidosis
- Associated features are nausea, vomiting, diarrhea and abdominal cramps.

Correct Answer. d

(26). A ventrolateral cordotomy is performed that produces relief of pain in the right leg. It is effective because it interrupts which of the
following?

a. Left dorsal column

b. Left ventrolateral spinothalamic tract

c. Right dorsal column

d. Right ventrolateral spinothalamic tract

Copyright © 2014 Delhi Academy of Medical Sciences, All Rights Reserved. 9/116
Solution. (b) Left ventrolateral spinothalamic tract
Ref: Read the text below
Sol:
Pain fibres cross over to the opposite side either at the level of entry into the spinal cord or one to two levels above and travel in the
lateral spinothalamic tract to the sensory cortex. A left ventrolateral cordotomy will disrupt the left lateral spinothalamic tract bringing
relief in the right leg.

Correct Answer. b

(27). The representational hemisphere is better than the categorical hemisphere in which of the following?

a. Language function

b. Recognition of objects by their form

c. Mathematical calculations

d. Understanding printed and spoken words

Solution. (b)
Ref: Read the text below
Sol:
- In 96% of right handed individuals (who constitute 91% of the population) the left hemisphere is “dominant”. In 70% of left handed
individuals, left hemisphere is dominant. Left hemisphere is also called “cateogorical”.
- Wernicke’s (for comprehension of printed/ written and spoken words) and Broca’s (for motor aspect of speech) areas of the left side are
better developed.
- Left hemisphere is concerned with categorization and symbolization.
- Frontal lobe lesions in the left hemisphere can cause ‘acalculia’ or inability to perform calculations.
- Right is the “non-dominant”. Also called “visuospatial” or “representational”.
- Right is specialized in visuospatial relations.
- Right hemisphere is concerned with identification of objects by their form and recognition of musical themes. It also plays a primary role
in recognition of faces.

Correct Answer. b

(28). Sperm maturation takes place in which organ?

a. Epididymis

b. Seminiferous tubules

c. Uterus

d. Vas deferens

Solution. (c) Uterus


Ref: Read the text below
Sol:
- Sperm maturation (acquisition of motility) also occur in epididymis, but the major maturation (capacitation) takes place in female genital
tract.
- If the female genital tract is provided in the options, it is the answer of choice, otherwise epididymis is the answer.

Correct Answer. c

(29). Ratio of motor units to number of muscle fibers in striated muscle of eyes is?

a. 1:3

b. 1:30

c. 1:300

d. 1:3000

Copyright © 2014 Delhi Academy of Medical Sciences, All Rights Reserved. 10/116
Solution. (a) 1:3
Ref: Read the text below
Sol:

Correct Answer. a

(30). Which of the following is not stored in cell-

a. Insulin

b. Cortisol

c. Thyroxin

d. Renin

Solution. (b) Cortisol


Ref: Read the text below
Sol:
- Cortisol is a steroid hormone.
- Steroid hormones are not stored in secretory vesicles before their secretion.

Correct Answer. b

(31). Pancreatic juice becomes thick if there is impaired-

a. Na secretion

b. Cl- secretion

c. NaH exchange

d. H+ secretion

Solution. (b) Cl- secretion


Ref: Read the text below
Sol:
In cystic fibrosis, CFTR is defective and there is impaired secretion of Cl- into the lumen resulting in no movement of water into the lumen
and pancreatic secretion becomes thick and viscous.

Correct Answer. b

(32). Function of synaptobrevin is?

a. Presynaptic vesicle fusion

b. Post synaptic vesicle fusion

Copyright © 2014 Delhi Academy of Medical Sciences, All Rights Reserved. 11/116
c. Inhibits synaptic transmission

d. Amplify synaptic transmission

Solution. (a) Presynaptic vesicle fusion


Ref: Read the text below
Sol:
- The primary role of SNARE proteins is to mediate vesicle fusion, that is , the exocytosis of cellular transport vesicles with the cell
membrane at the porosome or with a target compartment (such as a Lysosome).

Correct Answer. a

(33). A new dialysis technique is developed to clear circulating autoantibodies from plasma. The technique is based on using a special
semipermeable membrane. Which of the following is most likely to increase the rate of dialysis?

a. Decreasing the membrane pore size

b. Increasing thickness of the membrane

c. Increasing surface area of the membrane

d. Applying negative charge to the membrane

Solution. (c) Increasing surface area of the membrane


Ref: Read the text below
Sol :
- More the surface area of the membrane available, more will be the rate of diffusion across the membrane.

Correct Answer. c

(34). A 45- year old woman has hirsuitism, obesity, muscle wasting, and increased circulating levels of ACTH. The most likely cause of her
symptoms is

Copyright © 2014 Delhi Academy of Medical Sciences, All Rights Reserved. 12/116
a. Primary adrenocortical insufficiency (Addison disease)

b. Pheochromocytoma

c. Primary overproduction of ACTH (Cushing disease)

d. Treatment with exogenous glucocorticoids

Solution. (c) Primary overproduction of ACTH (Cushing disease)


Ref: Read the text below
Sol :

Correct Answer. c

(35). True statement about glucokinase is?

a. Km value is higher than normal blood sugar

b. Found exclusively in liver

c. G-6 P inhibit it

d. Has both G-6-phosphatase and pyruvate kinase activity

Solution. (a) Km value is higher than normal blood sugar


Ref.: Read the text below
Sol:
- Glucokinase is an an isoenzyme of hexokinase and is found in liver and cell of the pancreas.
- It has got low affinity (High km) for glucose and so is able to remove glucose from blood only when glucose is more than 100 mg/d.
- Glucose 6 phosphate has got no effect on action of glucokinase.
- Glucose 6 phosphase has got inhibitory effect on hexokinase activity.

Correct Answer. a

(36). In glycolysis, the first committed step is catalyzed by?

a. 2, 3 BPG kinase

b. Glucokinase

c. Hexokinase

Copyright © 2014 Delhi Academy of Medical Sciences, All Rights Reserved. 13/116
d. Phosphofructokinase

Solution. (d) Phosphofructokinase


Ref.: Read the text below
Sol:
- In glycolysis, PFK-1 converts fructose-6-phosphate to fructose—6-bisphosphate, which is destined only for glycolysis and no other
pathway.
- Action of hexokinase and glucokinase converts glucose to glucose-6-phosphate.
- Glucose 6 phosphate has got many additional fates also, in addition of entering in glycolysis. So, the step catalysed by
phosphofructokinase is the committed step of glycolysis.

Correct Answer. d

(37). Which of the following is an activator of LCAT?

a. Apo B-100

b. Apo B-48

c. Apo A IV

d. Apo A I

Solution. (d) Apo A I


Ref.: Read the text below
Sol:
- Lecithin cholesterol acyl transferase (LCAT) is the enzyme which is found on the surface of HDL and it transfers acyl moiety from a
phospholipids (lecithin) to the cholesterol and in this process lecithin is converted to lysolecithin, and cholesterol is converted to
cholesterol ester. Apo A1 acts as activating cofactor for LCAT enzyme.

Correct Answer. d

(38). During prolonged starvation, the rate of gluconeogenesis depends on

a. Increased alanine levels in liver

b. Decreased cGMP in liver

c. ADP in liver

d. Decreased essential fatty acids in liver

Solution. (a) Increased alanine levels in liver


Ref: Read the text below
Sol:
- In the fasting state, there is a considerable output of alanine from skeletal muscle, far in excess of its concentration in the muscle
proteins that are being catabolized.
- It is formed by transamination of pyruvate produced by glycolysis of muscle glycogen, and is exported to the liver.
- In the liver after transamination alanine is converted back to pyruvate, which is a substrate for gluconeoenesis.

Correct Answer. a

(39). The biosynthesis of the enzyme pyruvate carboxylase is repressed by?

a. Insulin

b. Cortisol

c. Glucagon

d. Epinephrine

Solution. (a) Insulin


Ref: Read the text below
Sol:
- Pyruvate carboxylase is gluconeogenic hormone.
- Insulin inhibits gluconeogenesis and has got inhibitory effect on pyruvate carboxylase.

Correct Answer. a

Copyright © 2014 Delhi Academy of Medical Sciences, All Rights Reserved. 14/116
(40). Rate limiting enzyme of cholesterol biosynthesis is?

a. HMG Co A reductase

b. HMG Co A synthase

c. 7 alpha hydroxylase

d. Phosphofructokinase

Solution. (a) HMG Co A reductase


Ref: Read the text below
Sol:
- HMG Co A reductase is the rate limiting enzyme for cholesterol synthesis.
- HMG Co A reductase enzyme is both under covalent modification as well as under allosteric regulation.
- Insulin and thyroid hormone increases the activity of HMG Co A reductase and mevalonate, cholesterol, glucagon, glucocorticoids inhibit
the activity of HMG Co A reductase enzyme.

Correct Answer. a

(41). All of the following statement about lipoprotein lipase is true, except?

a. Found in adipose tissue

b. Found in myocytes

c. Deficiency leads to hypertriacylglyserolemia

d. Does not require CII as a cofactor

Solution. (d) Does not require CII as a cofactor


Ref: Read the text below
Sol:
- Lipoprotein lipase is extracellular enzyme adhered to the endothelial cell of blood vessels supplying various organs.
- Heparan sulphate is required to adhere this enzyme to the endothelial cell.
- High density of negative charges in the heparin sulphate bring the positive charged molecule of lipoprotein lipase in the vicinity and
holds them by electrostatic interaction.
- Lipoprotein lipase activity is stimulated by Apolipoprotein CI and CII.
- So triacyglycerol of VLDL and CM is acted upon by this enzyme and fatty acid is send to the organ.

Correct Answer. d

(42). Regarding synthesis of triacylglycerol in the adipose tissue, all of the following are true except?

a. Synthesis from dihydroxyacetone phosphate

b. Glycerol kinase enzyme plays an important role

c. Glycerol 3 phosphate dehydrogenase plays an important role

d. Phosphatidate is hydrolysed

Solution. (b) Glycerol kinase enzyme plays an important role


Ref: Read the text below
Sol:
- Adipose tissue does not possess glycerol kinase enzyme, so for the synthesis of triacylglyserol, adipose tissue is dependent on glycolysis
which provides dihydroxyacetone phosphate which gets converted to Glycerol 3 phosphate with the help of enzyme Glycerol 3 phosphate
dehydrogenase which catelyses a reversible reaction.
- Now this glycerol 3 phosphate acts as a nucleus on which fatty acid gets esterified to form triacylglyserol.

Correct Answer. b

(43). Hypolipidemic agents acts on?

a. HMG Co A synthetase

b. HMG Co A reductase

c. HMG Co A mutase

d. HMG Co A lyase

Copyright © 2014 Delhi Academy of Medical Sciences, All Rights Reserved. 15/116
Solution. (b) HMG Co A reductase
Ref: Read the text below
Sol:
- Hypolipidemic drugs (statin) when given to reduce cholesterol content in the circulation, do so via inhibiting endogenous synthesis of
cholesterol by acting on HMG Co A reductase enzyme.

Correct Answer. b

(44). Which biochemical reaction does not occur in mitochondria?

a. Kreb’s cycle

b. Urea cycle

c. Gluconeogenesis

d. Fatty acid synthesis

Solution. (d) Fatty acid synthesis


Ref: Read the text below
Sol:
- Fatty acid synthesis occurs in the cytoplasm.
- In cytoplasm there occurs fatty acid synthase complex, which is a dimer and where occurs fatty acid synthesis.
- Kreb’s cycle occurs entirely within the mitochondria.
- Gluconeogenesis and urea cycle occurs partly in mitochondria and partly in cytoplasm.

Correct Answer. d

(45). One week infant, having classic phenylketonuria, which of the following statement is correct?

a. Tyrosine is nonessential amino acid for this infant

b. High levels of phenylpyruvate is appearing in the urine

c. Therapy must begin within first year of life

d. Diet therapy should be discontinued once infant reaches adulthood.

Solution. (b) High levels of phenylpyruvate is appearing in the urine


Ref: Read the text below
Sol:
- Tyrosine becomes essential amino acid for this infant as phenylalnine can not be converted to tyrosine.
- Phenylalanine is converted to phenylpyruvate, phenylacetate and phenyllactate in this disorder.
- Therapy of phenylalanine restricted diet should be started within 7 to 10 days of life to avoid mental retardation.
- Adult PKU patient shows deterioration of attention and thinking process after discontinuation of diet, so life long restriction of dietary
phenylalanine is recommended.

Correct Answer. b

(46). In which one of the following tissue glucose transports in the cell is enhanced by insulin?

a. Brain

b. Lens

c. RBC

d. Adipose tissue

Solution. (d) Adipose tissue


Ref.: Read the text below
Sol:
- Major tissues in which glucose transport require insulin are muscle and adipose tissue.
- The metabolism in the liver responds to insulin, but hepatic glucose transport is determined by blood glucose concentration, and dose
not require insulin.

Correct Answer. d

(47). Squalene is the intermediate product during synthesis of?

Copyright © 2014 Delhi Academy of Medical Sciences, All Rights Reserved. 16/116
a. VLDL

b. Cholesterol

c. Tachysterol

d. Lanosterol

Solution. (b) Cholesterol


Ref: Read the text below
Sol:
- Squalene is 30 carbon structure which is formed as an intermediate during synthesis of cholesterol.
- From squalene cholesterol is formed after removal of 3 CH3 group.

Correct Answer. b

(48). Immediate precursor in the production of acetoacetate is?

a. Acetoacetyl Co A

b. Hydroxyl methyl glutaryl Co A

c. Malonyl Co A

d. Iso valeric acid

Solution. (b) Hydroxyl methyl glutaryl Co A


Ref: Read the text below
Sol:
- HMG Co A is the immediate precursor of acetoacetate which is the 1st ketone body formed.
- β-(OH) butyrate is formed once β–(OH) butyrate dehydrogenase acts on acetoacetate.
- β-(OH) butyrate is most abundant (predominant) ketone body formed.

Correct Answer. b

(49). LDL receptor in liver can be detected by?

a. Apo B 100 and apo E

b. Apo B 100 and apo A

c. Apo E

d. Apo E and apo A

Solution. (a) Apo B 100 and apo E


Ref: Read the text below
Sol:
- LDL receptors are present on the liver and are involved in removal of LDL particle formed in the circulation.
- They recognize Apo B-100 and Apo E.
- So LDL receptors are not only involved in removal of LDL particles having Apo B-100, they also can remove chylomicron remnant having
ApoE.

Correct Answer. a

(50). If choline moiety is replaced by ethanolamine, the net product is?

a. Cerebroside

b. Sphingomyelin

c. Cephalin

d. Plasminogen

Solution. (c) Cephalin


Ref.: Read the text below
Sol:
- Phosphatidylethanolamine is also known as cephalin.
- Phosphatidylcholine is also known as lecithin.

Correct Answer. c

Copyright © 2014 Delhi Academy of Medical Sciences, All Rights Reserved. 17/116
(51). Most common organ involved in primary amyloidosis is

a. heart

b. liver

c. kidney

d. spleen

Solution. C
Reference: Robbins and Cotran Pathological Basis of Disease, Harrison’s Principles of Internal Medicine 20th edition
The most common organ involved in primary and secondary amyloidosis is kidney. The most common cause of death in primary
amyloidosis is heart.

Correct Answer. c

(52). Which of the following dyads of stains is wrongly matched?

a. Sweet’s Reticulin- Cirrhosis

b. Von Kossa- Calcium

c. Verhoeff Van Geison- Mucin

d. Rhodanine- Copper

Solution. C
Verhoeff Van Geison is the special stain for elastin

Correct Answer. c

(53). Verocay bodies are seen in

a. Neurofibroma

b. Schwannoma

c. Malignant peripheral nerve sheath tumor

d. Pilocytic astrocytoma

Solution. B
Microscopically, they are comprised of an admixture of dense and loose areas referred to as Antoni A and Antoni B areas, respectively.
The dense eosinophilic Antoni A areas often contain spindle cells arranged into cellular intersecting fascicles. Palisading of nuclei is
common and “nuclear-free zones” that lie between the regions of nuclear palisading are termed Verocay bodies

Correct Answer. b

(54). All are features of necrosis except

a. Adjacent inflammation is brisk

b. Membrane damage

c. Karyolysis

d. Can be physiological

Copyright © 2014 Delhi Academy of Medical Sciences, All Rights Reserved. 18/116
Solution. D

Correct Answer. d

(55). All are true about IgA nephropathy except

a. Due to an abnormally glycosylated IgA1

b. CD 71 is the IgA1 receptor on mesangial cells

c. C1q deposits are seen in the mesangium

d. Within 1-3 days of a non specific respiratory infection

Solution. C
An abnormally glycosylated IgA1 (i.e., galactose-deficient IgA1 [Gd-IgA1]) immunoglobulin is thought to play a central role in the
pathogenesis. This abnormal IgA may elicit an autoimmune response, and autoantibodies may form large immune complexes with
circulating IgA. These complexes deposit in the glomerular mesangium; this unusual location may be related to physicochemical features
of the IgA and may be facilitated by an IgA1 receptor (CD71) on mesangial cells. The presence of C3 in the mesangium and the absence of
C1q and C4 points to activation of the alternative complement pathway in the pathogenesis

Correct Answer. c

(56). The following microscopic appearance seen on kidney biopsy suggests the following gross appearance of kidney

a. Leather grain appearance

b. Flea bitten appearance

c. Irregularly scarred kidneys

d. Enlarged waxy kidney

Copyright © 2014 Delhi Academy of Medical Sciences, All Rights Reserved. 19/116
Solution. A
Grossly, the kidneys are symmetrically atrophic. Typically the renal surface shows diffuse, fine granularity that resembles grain leather.
Microscopically, the most prominent change is hyaline thickening of the walls of the arterioles, known as hyaline arteriolosclerosis. This
appears as a homogeneous, pink hyaline thickening, at the expense of the vessel lumina, with loss of underlying cellular detail.

Correct Answer. a

(57). Intrinsic pathway of apoptosis involves

a. Caspase 3

b. Caspase 6

c. Caspase 9

d. Caspase 8

Solution. C
The two initiating pathways converge to a cascade of caspase activation, which mediates the final phase of apoptosis. The mitochondrial
pathway leads to activation of the initiator caspase-9, and the death receptor pathway to the initiator caspases-8 and -10. After an initiator
caspase is cleaved to generate its active form, the enzymatic death program is set in motion by rapid and sequential activation of the
executioner caspases. Executioner caspases, such as caspase-3 and -6, act on many cellular components. For instance, these caspases,
once activated, cleave an inhibitor of a cytoplasmic DNase and thus make the DNase enzymatically active; this enzyme induces cleavage
of DNA. Caspases also degrade structural components of the nuclear matrix and thus promote fragmentation of nuclei.

Correct Answer. c

(58). Grade Group system of histological grading by WHO 2016 has been given for which of the following cancers?

a. Prostate cancer

b. Renal cell cancer

c. Bladder cancer

d. Breast cancer

Solution. A
Over the years, the Gleason grading system has undergone several changes. Currently, Gleason total scores 2–5 are no longer assigned
and in practice the lowest total score is now assigned a 6, although the scale continues to range from 2 to 10. This leads to a logical yet
incorrect assumption on the part of patients that their Gleason 6 cancer is in the middle of the scale, triggering the fear that their cancer
is serious and the assumption that treatment is necessary despite Gleason score 6 actually being favorable risk. To address these issues, a
new 5-grade group system has been developed:
Grade Group 1 (Gleason score ≤6)
Grade Group 2 (Gleason score 3+4 = 7)
Grade Group 3 (Gleason score 4+3 = 7)
Grade Group 4 (Gleason score = 8)
Grade Group 5 (Gleason scores 9 and 10)

Correct Answer. a

(59). All are true about seminomas except

a. Beta hcg positive

b. AFP positive

c. Chemosensitive

d. Radiosensitive

Solution. B
GCTs are either seminomas or nonseminomas. For a tumor to be considered a seminoma, it must be 100% seminoma. Any mixed GCT is

best approached as a nonseminomatous GCT (NSGCT). Seminomas represent of cases. Seminomas arise most commonly in
patients in the fourth decade of life. Seminomas may contain syncytiotrophoblastic cells which may secrete β human chorionic
gonadotropin (HCG). Seminomas do not secrete α fetoprotein (AFP). Seminomas are exquisitely sensitive to both chemotherapy and
radiation therapy.

Correct Answer. b

Copyright © 2014 Delhi Academy of Medical Sciences, All Rights Reserved. 20/116
(60). Dermoids undergo malignant transformation most commonly into

a. Melanoma

b. Thyroid carcinoma

c. Squamous cell carcinoma

d. Adenocarcinoma

Solution. -NA-

Correct Answer. c

(61). Richter transformation of CLL is associated with

a. p53 mutations

b. NOTCH1 mutations

c. Loss of CDKN2A

d. All of the above

Solution. D
DLBCL transformation is associated with TP53 and NOTCH1 mutations, CDKN2A deletions, and MYC translocations

Correct Answer. d

(62). Most common site of MALToma is

a. Ocular adnexa

b. Ileum

c. Stomach

d. Salivary gland

Solution. C
The stomach is the most common site of MALT lymphoma, affected in 35% of all cases. Other common sites include the eyes and ocular
adnexa (affected in 13% of cases), skin (9%), lungs (9%), salivary glands (8%), breasts (3%), and thyroid (2%)

Correct Answer. c

(63). The distinction of leiomyosarcoma from leiomyoma is based on all of the following features except

a. Necrosis

b. Mitotic activity

c. Nuclear atypia

d. Size of the lesion

Solution. D
The distinction from leiomyoma is based on nuclear atypia, mitotic index, and zonal necrosis.

Correct Answer. d

Copyright © 2014 Delhi Academy of Medical Sciences, All Rights Reserved. 21/116
(64). Following finding in a mass is indicative of

a. Hyperplasia

b. Metaplasia

c. Dysplasia

d. Anaplasia

Solution. D
Mitoses are indicative of rapid cell growth. Hence, cells in mitosis are often seen in normal tissues exhibiting rapid turnover, such as the
epithelial lining of the gut and nonneoplastic proliferations such as hyperplasias. More important as a morphologic feature of malignancy
are atypical, bizarre mitotic figures, sometimes with tripolar, quadripolar, or multipolar spindles.

Correct Answer. d

(65). Hereditary pancreatic carcinoma is classically associated with which of the following

a. STK11

b. BRCA2

c. PRSS1

d. All of the above

Solution. D

Correct Answer. d

Copyright © 2014 Delhi Academy of Medical Sciences, All Rights Reserved. 22/116
(66). The following technique can help identify which of the following genetic changes

a. Deletion

b. Translocation

c. Amplification

d. All of the above

Solution. D
Fluorescent in situ hybridisation can demonstrate deletion/monosomy, trisomy (as pictured), translocations, fusions and amplifications.

Correct Answer. d

(67). All of the following disorders occur due to trinucleotide repeats in non coding areas except

a. Fragile X syndrome

b. Friedrich ataxia

c. Huntington disease

d. Myotonic dystrophy

Solution. C

Correct Answer. c

Copyright © 2014 Delhi Academy of Medical Sciences, All Rights Reserved. 23/116
(68). Molecular subtyping of breast carcinoma is based on

a. ER, PR and Her 2 neu status

b. Ki 67 index

c. Gene expression profiling

d. Cytogenetic analysis

Solution. C
Recent detailed description of genomic alterations and gene and protein expression in large cohorts of breast cancers has provided a
framework for a molecular classification.

Correct Answer. c

(69). Diffuse gastric cancer is associated with

a. CDH1 mutations

b. BRCA2 mutations

c. p53 mutations

d. All of the above

Solution. D
Diffuse gastric cancer with signet ring cell appearance can be sporadic or familial and is associated with above mentioned genes

Correct Answer. d

(70). Transepithelial transport is affected in all of the following malabsorption syndromes except

a. Celiac disease

b. Whipple disease

c. Inflammatory bowel disease

d. Abetalipoproteinemia

Solution. C

Correct Answer. c

(71). Favourable prognostic indicators in neuroblastoma except

Copyright © 2014 Delhi Academy of Medical Sciences, All Rights Reserved. 24/116
a. Age <18 months

b. Schwannian stroma

c. N- myc amplification

d. TRKA expression

Solution. C

Correct Answer. c

(72). The following mutation if homozygous will result in

a. Cooley’s anemia

b. Sickle haemoglobin

c. Beta thalassemia minor

d. No disease

Solution. A
Homozygous beta 0 globin mutations will result in absent beta globin production and thus lead to beta thalassemia major (Cooley’s
anemia)

Correct Answer. a

(73). Most common gene mutated in adenocarcinoma of lung

a. KRAS

b. P53

Copyright © 2014 Delhi Academy of Medical Sciences, All Rights Reserved. 25/116
c. EGFR

d. ALK

Solution. A
Most commonly mutated gene in adenocarcinoma both Lung and Pancreas- KRAS

Correct Answer. a

(74). Collapsing glomerulopathy is seen in

a. Sickle cell anemia

b. Goodpasture syndrome

c. HIV

d. Heroin abuse

Solution. C
Collapsing type of FSGS is characteristic of HIV and Pamidronate

Correct Answer. c

(75). The least affected organ in graft vs host disease is

a. Liver

b. Skin

c. Lung

d. Gastrointestinal tract

Solution. C
GVHD classically affects skin (most commonly), liver and GIT

Correct Answer. c

(76). A 56 yr old man, Amish with heart failure is to be treated with a diuretic drug. Drugs A and B have same mechanism of action. Drug A in
dose of 50 mg produces the same magnitude of dieresis as 500 mg of drug B. This suggest that

a. Drug B is less efficacious than drug A

b. Drug A is more potent then drug B

c. Drug A is safer drug the drug B

d. Drug A will have shorter duration of action than drug

Solution. (b) Drug A is more potent then drug B


Ref: Read the text below
Sol:
- The drug producing the same response at lower dose is more potent.
- Drug A is 10 times more potent than drug B.
- This data does not indicate anything about efficacy, safety or duration of action.

Correct Answer. b

(77). A 30 year old patient on digoxin therapy has developed digitalis toxicity. The plasma digoxin level is 4ng/ml. renal function is normal and
the plasma t1/2 for digoxin in this patient is 1.6 days. How long should you withhold digoxin in order to reach a safer yet probably
therapeutic of 1 ng/ml?

a. 1.6 days

b. 2.4 days

c. 3.2 days

d. 4.8 days

Copyright © 2014 Delhi Academy of Medical Sciences, All Rights Reserved. 26/116
Solution. (c) 3.2 days
Ref: Read the text below
Sol:
- We want decrease the plasma concentration of digoxin from 4ng/ml to 1ng/ml.
- It will take two half lives.
- Thus time required will be 2×t1/2i.e. 2×1.6=3.2 days.

Correct Answer. c

(78). Atropine is useful in organophosphate poisoning because it

a. Reactivates acetyl cholinesterase

b. Competes with acetylcholine release

c. Binds with both nicotinic and muscarinic acetylcholine receptors

d. Is a competitive antagonist of acetylcholine

Solution. (d) Is a competitive antagonist of acetylcholine


Ref: Read the text below
Sol:
- Atropine acts as an antagonist at muscarinic receptors.
- It has no activity on nicotinic receptors and do not interfere with the release of ACh.

Correct Answer. d

(79). A drug ‘X’ is a selective α1A receptor blocker that affords symptomatic relief without producing significant hypotension in patients with
benign hyperplasia of prostate. X is most likely to be

a. Terazosin

b. Doxazosin

c. Prazosin

d. Tamsulosin

Solution. (d) Tamsulosin


Ref: Read the text below
Sol:

Correct Answer. d

(80). A 60 years old male with difficulty in urination was found to have enlarged prostate and blood pressure of 160/100 mmHg. The drug that
should be used in this case is

a. Doxazosin

Copyright © 2014 Delhi Academy of Medical Sciences, All Rights Reserved. 27/116
b. Labetalol

c. Propranolol

d. Phentolamine

Solution. (a) Doxazosin


Ref: Read the text below
Sol:
- Doxazosin is an alpha-adrenergic blockers. It relaxes your veins and arteries so that blood can more easily pass through them. It also
relaxes the muscles in the prostate and bladder neck, making it easier to urinate.
- Doxazosin is used to treat hypertension(high blood pressure), or to improve urination in men with benign prostatic hyperplasia (enlarged
prostate).
- The extended-release form of doxazosin (Cardura XL) is for use only in treating benign prostatic hyperplasia and should not be used to
treat hypertension.

Correct Answer. a

(81). Which of the following drugs inhibit 5’ –deiodinase?

a. Propylthiouracil

b. Methimazole

c. Lugol’s iodine

d. Radioactive iodine

Solution. (a) Propylthiouracil


Ref: Read the text below
Sol:

Correct Answer. a

(82). The unique property of SERMs is that they

a. Have both estrogenic and progestational agonistic activity

b. Inhibits the aromatase enzyme that is required for estrogen synthesis

c. Produces estrogenic effect without binding to estrogen receptors

d. Act as agonist in some tissue and antagonist in other tissues

Solution. (d) Act as agonist in some tissue and antagonist in other tissues
Ref: Read the text below
Sol:
- AERMs are drugs with agonistic action on estrogen receptors in some tissue and antagonistic action in other tissues e.g. temoxifen,
raloxifene.

Correct Answer. d

(83). Inverse agonist of benzodiazepine receptor is

a. Phenobarbitone

b. Flumazenil

c. Beta carboline

d. Gabapentin

Copyright © 2014 Delhi Academy of Medical Sciences, All Rights Reserved. 28/116
Solution. (c) Beta carboline
Ref: Read the text below
Sol:

Correct Answer. c

(84). Ritonavir inhibits all of the following except

a. Amiodarone

b. Cisapride

c. Phenytoin

d. Midazolam

Solution. (c) Phenytoin


Ref: Read the text below
Sol:
- Ritonavir induces CYP 1A2 and inhibits the major P450 isoforms (3A4 and 2D6).
- Amiodarone – decreased metabolism, possible toxicity
- Midazolam and triazolam – contraindicated
- Carbamazepine – decreased metabolism, possible toxicity
- Cisapride – decreased metabolism, possible prolongation of Q-T interval and life-threatening arrythmias
- Disulfiram (with ritonavir oral preparation) – decreased metabolism of ritonavir
- voriconazole – ritonavir increases metabolism of voriconazole
- Because of potential toxicities, ritonavir should not be used concurrently with various antiarrhythmics (amiodarone, encainide, lecainide,
quinidine) and highly metabolized sedative/hypnotics (i.e., alprazolam, diazepam, lurazepam, midazolam, and triazolam).

Correct Answer. c

(85). Choose the most inappropriate statement regarding “Iodine use in thyroid disorder”:

a. Causes Iodism

b. Contraindicated in Hyperthyroidism

c. Inhibit Formation of Iodo Thyronine

d. Thyroxine Release

Copyright © 2014 Delhi Academy of Medical Sciences, All Rights Reserved. 29/116
Solution. (b) Contraindicated in Hyperthyroidism
Ref: Read the text below
Sol:
- Iodine has several effects on thyroid function. In hyperthyroid patients, iodine acutely inhibits hormonalsecretion within hours, but the
responsible mechanisms are uncertain. This is the most acute effect ofiodine on thyroid status, occurring within one to two days of the
start of therapy.
- A second effect involves inhibition of thyroid hormone synthesis. In normal subjects, the administrationof pharmacologic amounts of
iodine leads to temporary inhibition of iodine organ fication in the thyroid gland, thereby diminishing thyroid hormone biosynthesis, a
phenomenon called the Wolff-Chaikoff effect
- However, within two to four weeks of continued exposure to excess iodine, organification and thyroid hormone biosynthesis resume in a
normal fashion. This is called escape from the Wolff-Chaikoff effect.

Correct Answer. b

(86). A pharmacy fellow is trying to ascertain the plasma concentration of an experimental anti-arrhythmic agent (Drug X) at steady-state.A
persistent intravenous infusion of the agent began 6 hours earlier at a rate of 3 mg/min.Drug X has a half-life of 3 hours, a volume of
distribution of 120 L and a clearance of 0.6 L/min.What will be the plasma concentration at steady-state if the rate of infusion remains
constant?

a. 0.005 mg/L

b. 0.4 mg/L

c. 2.0 mg/L

d. 5.0 mg/L

Solution. (d) 5.0 mg/L


Ref: Read the text below
Sol:

Correct Answer. d

(87). A diabetic man aged 55 years is brought to the emergency room in an unresponsive state. The laboratory values obtained are listed
below:
- PCO2 19 mm Hg
- HCO3 11 mEq/L
- pH 6.9.
What is the most suitable immediate treatment of this patient?

a. Administration of an oral hypoglycemic agent

b. Administration of bicarbonate

c. Administration of insulin

d. Close observation only

Copyright © 2014 Delhi Academy of Medical Sciences, All Rights Reserved. 30/116
Solution. (c) Administration of insulin
Ref: Read the text below
Sol: This patient is in a diabetic keto acidotic coma. The objectives in treating such a patient include:
- To increase the rate of glucose utilization by insulin-dependent tissues
- To reverse ketonemia and acidosis
- To replenish fluid imbalances
Commonly prescribed oral hypoglycemic agents are for the maintenance of NIDDM patients
It would not be appropriate in an acute setting.
Transient elevation of pH is cause of treatment with bicarbonate

Correct Answer. c

(88). You are considering a cyclic peptide antibiotic Capreomycin as a second line for the management of MDR-TB.Which of the following is not
a likely consequence?

a. Ototoxicity

b. Renal toxicity

c. Hypomagnesemia

d. Hyperkalemia

Solution. (d) Hyperkalemia


Ref: Read the text below
Sol:
- A cyclic peptide antibiotic derived from Streptomyces capreolus is Capreomycin It is an important second-line agent used for treatment
of MDR-TB.
- Side effects of capreomycin are relatively common.
- Significant hypokalemia, hypomagnesemia as well as oto and renal toxicity have been reported.

Correct Answer. d

(89). A man aged 54 years is treated with lidocaine to regulate his cardiac rhythm. Which of the following represents the Mechanism of Action
of this medication?

a. Beta adrenergic receptor blocker

b. Class 1A sodium channel blocker

c. Class 1B sodium channel blocker

d. Potassium channel blocker

Copyright © 2014 Delhi Academy of Medical Sciences, All Rights Reserved. 31/116
Solution. Ref: Read the text below
Sol: A class 1B sodium channel blocker is Lidocaine
It is selective to ischemic tissue It specially activates fast sodium channels in the bundle of His, Purkinje fibers and ventricular
myocardium. Beta adrenergic receptor blockers include
- Propranolol
- Esmolol
- Timolol
- Metoprolol
As antiarrhythmic agents-
- they decrease cAMP levels
- decrease calcium currents
- suppress abnormal pacemakers.
Calcium channel blockers include:
- Verapamil
- Diltiazem
- Bepridil
They are used in the prevention of nodal arrhythmias. They decrease conduction velocity and increase the PR interval. Class 1B sodium
channel blockers include:
- Lidocaine
- Mexiletine
- Tocainide
They act to depress the action potential and stabilize the cell membrane. They are used in acute ventricular arrhythmias and digitalis-
induced arrhythmias. Class 1C sodium channel blockers include
- Flecainide
- Encainide
- Propafenone
They depress the action potential at phase 0 and stabilize cell membranes. As a last resort they are used in refractory tachyarrhythmias
due to their side effects. Note:
Amiodarone
- It blocks inactivated Na+ channels
- It has a relatively rapid rate of recovery from block.
- It also decreases Ca2+ current and transient outward delayed-rectifier and inward rectifier K+ currents and exerts a noncompetitive
adrenergic blocking effect. In most tissues, prolongs APD Amiodarone potently inhibits abnormal automaticity. Amiodarone decreases
conduction velocity by Na+ channel block. Prolongations of the PR, QRS, and QT intervals and sinus bradycardia are frequent during
chronic therapy. Amiodarone prolongs refractoriness in all cardiac tissues Na+ channel block, delayed repolarization owing to K+
channel block and inhibition of cell–cell coupling all may contribute to this effect.

Correct Answer. c

(90). Which of the following represents the Mechanism of Action of Metronidazole?

a. Cell wall synthesis inhibition

b. Inhibition of protein synthesis

c. Disruption of cell membrane

d. DNA damage

Solution. (d) DNA damage


Ref: Read the text below
Sol: Generation of intracellular reactive intermediates that damage DNA by electron transfer system is the action of Metronidazole.
Cell wall synthesis inhibitors are-
- penicillins
- cephalosporins
- Vancomycin and bacitracin.
Drugs affecting protein synthesis include:
- Lincosamides aminoglycosides
- Streptogramins
- Macrolides
- Chloramphenicol
- Tetracyclins
Disruption of cell membrane is caused by-
- Polymixins
- Daptomycin

Correct Answer. d

(91). Naloxone is contraindicated in neonatal resuscitation if the mother is on

a. Cocaine

b. Amphetamine

Copyright © 2014 Delhi Academy of Medical Sciences, All Rights Reserved. 32/116
c. Methadone

d. Phencyclidine

Solution. (c) Methadone


Ref: Read the text below
Sol:
- When the mother is on opioids, fetus becomes opioid dependent in-utero and the use of naloxone in respiratory resuscitation may result
in severe withdrawal symptoms.

Correct Answer. c

(92). The difference between iron sorbitol-citric acid and iron dextran is that the former

a. Cannot be injected i.v.

b. Is not bound to transferring in plasma

c. Is not excreted in urine

d. Produces fewer side effects

Solution. (a) Cannot be injected i.v.


Ref: Read the text below
Sol:

Correct Answer. a

(93). Which of the following is not an adverse effect of chronic amiodarone therapy?

a. Pulmonary fibrosis

b. Hypothyroidism

c. Hyperthyroidism

d. Systemic lupus erythematosis

Solution. (d) Systemic lupus erythematosis


Ref: Read the text below
Sol:
ADVERSE EFFECT OF CHRONIC AMIODARONE THERAPY

Correct Answer. d

Copyright © 2014 Delhi Academy of Medical Sciences, All Rights Reserved. 33/116
(94). A patient was started on antihypertensive medications and he developed renal failure. The drug most likely to cause ARF is

a. Beta-blocker

b. Alpha-blocker

c. Calcium channel blocker

d. ACE inhibitor

Solution. (d) ACE inhibitor


Ref: Read the text below
Sol:
- ARF (Acute renal failure) is precipitated by ACE inhibitors in patients with bilateral renal artery stenosis due to dilation of efferent
arterioles and fall in glomerular filtration pressure.
- So, ACE inhibitors are contraindicated in such patients.

Correct Answer. d

(95). A patients Aniket has been taking digoxin for several years and is about to receive atropine for some other indication. A common side
effect of digoxin that can be blocked by atropine is

a. Decreased appetite

b. Increased cardiac contractility

c. Increased PR interval on the ECG

d. Tachycardia

Solution. (c) Increased PR interval on the ECG


Ref: Read the text below
Sol:
- Digitalis possesses vagomimetic action and can cause bradycardia and decreased AV conduction.
- This latter affect is manifested in the ECG as increased PR interval.
- Atropine can block this adverse effect.

Correct Answer. c

(96). Most common cause of pyomyositis is:

a. Streptococcus pyogenes

b. Pseudomonas aeruginosa

c. Staphylococcus aureus

d. E. Coli

Solution. C
Pyomyositis is an unusual infection of skeletal muscles that is seen primarily in tropical climates but also occursin immunocompromised
and HIV-infected patients. Pyomyositis presents as fever, swelling, and pain overlyingthe involved muscle.
Tropical pyomyositis – MC cause is S. aureus, (acute bacterial myositis – MC cause is Group A Streptococcus) (Overall - MC cause of
pyomyositis is S.aureus).

Correct Answer. c

(97). Which of the following is NOT a source of infection in plague?

a. Case of bubonic plague

b. Case of pneumonic plague

c. Infected rodents

d. Infected rat fleas

Solution. A
MC source for bubonic plague is- Infected rat fleas followed by Infected rodents.
Source for pneumonic plague is- Case of pneumonic plague.

Correct Answer. a

Copyright © 2014 Delhi Academy of Medical Sciences, All Rights Reserved. 34/116
(98). Tabes dorsalis is seen in:

a. Primary syphilis

b. Secondary syphilis

c. Tertiary syphilis

d. Latent syphilis

Solution. C
Tabes dorsalis is seen in Tertiary syphilis.

Correct Answer. c

(99). Sputum can be disinfected by all except:

a. Autoclaving

b. Boiling

c. Cresol

d. Chlorhexidine

Solution. D
The commonest pathogen suspected to be present in sputum- Tubercle bacilli. So it has to be disinfected by anappropriate technique.
Chorhexidine is a skin antiseptic, used for burns or hand disinfection. It is not mycobactericidal.
Sputum can be disinfected by:
Autoclaving – Ideal method.
Other methods include-Burning of the material, Cresol 5% or Phenol or Boiling.

Correct Answer. d

(100). Recommended transport medium for stool specimensuspected to contain enteric pathogens is:

a. Amie’s medium

b. Buffered glycerol saline medium

c. MacConkey medium

d. Blood agar

Solution. B
Recommended transport medium for stool specimen is-
Buffered glycerol saline medium (when Salmonella or Shigella is suspected).
VR medium-(when Vibrio cholerae is suspected).

Correct Answer. b

(101). Skin test based on neutralization reaction is/are:

a. Casoni test

b. Lepromin test

c. Tuberculin test

d. Schick test

Solution. D
Neutralization is a reaction in which antigens and antibodies neutralize each other.
The reaction is used to identify toxins and antitoxins, as well as viruses and viral antibodies.

Correct Answer. d

(102). Neil mosser reaction or tunica reaction is useful to differentiate between:

Copyright © 2014 Delhi Academy of Medical Sciences, All Rights Reserved. 35/116
a. R. prowazekii and R. typhi

b. R. typhi and R. rickettsii

c. R. prowazekii and R. rickettssii

d. R. rickettsia and C. burnetti

Solution. A
Neilmooser reaction is done to differentiate epidemic typhus (R. Prowazekii) which gives a negative tunica reaction and endemic typhus
(R. typhi) which gives a positive tunica reaction.

Correct Answer. a

(103). Schizonts are not seen in peripheral smear in which type ofmalaria?

a. P. vivax

b. P. falciparum

c. P. ovale

d. P. malariae

Solution. B
After the ring form is formed in the peripheral blood, the remaining of RBC cycle of P.falciparum occurs in the capillaries of brain and
internal organs. Hence, only the ring forms are found in the peripheral blood by microscopic examination but not late trophozoites and
schizonts. However, for other Plasmodium species, all erythrocytic stages occur in peripheral blood vessels.

Correct Answer. b

(104). Congenital toxoplasmosis false is:

a. IgA is better than IgM in detection

b. Diagnosed by detection of IgM in cord blood

c. Dye test is gold standard & it detects IgG

d. Avidity test must be done to differentiate IgA & IgM

Solution. D
Avidity test is done for IgG to differentiate recent and past infection. Avidity low- indicates recent infection,
Aviditystrong- indicates past infection.
Congenital toxoplasmosis is diagnosed:
–– By detection of IgM in cord blood.
–– IgA is experimental but better sensitivity than IgM for diagnosis of congenital toxoplasmosis.
–– Sabin Feldman Dye test is gold standard method for diagnosis of toxoplasmosis, however it detects IgG, hencecannot differentiate
recent from past infection.
Test to differentiate recent and past infection:
–– IgG Avidity test - Avidity low- indicates recent infection, Avidity strong- indicates past infection.
–– IgM detection by ELISA or IFA- indicates recent infection.
–– Detection of Toxoplasma antigen or genes.

Correct Answer. d

(105). Man acts as definitive & intermediate host for:

a. Taenia solium- Cysticercosis

b. Taenia solium- Intestinal taeniasis

c. Taenia saginata- Intestinal taeniasis

d. Taenia saginata- Cysticercosis

Solution. A

Correct Answer. a

(106). Arc-5 in CIEP is diagnostic for:

Copyright © 2014 Delhi Academy of Medical Sciences, All Rights Reserved. 36/116
a. Hydatid disease

b. Cysticercosis

c. Cryptococcosis

d. brucellosis

Solution. A
Immunodiffusion and counter current electro phoresis (CIEP) are used for hydatid disease- detecting antibodyagainst antigen-5 (arc-5).
Western blot- detecting antibody against antigen B fragment.
Antibody methods are useful for sero-epidemiological study but cannot differentiate recent and past infection.

Correct Answer. a

(107). Which of the following is viviparous?

a. Strongyloides stercoralis

b. Trichinella spiralis

c. Enterobius

d. Ascaris

Solution. B
Viviparous- worms that lay Larva directly, without egg formation- E.g.Filarial worms, Trichinella & Dracunculus

Correct Answer. b

(108). Eggs of all the following helminthic worms float in asaturated salt solution EXCEPT:

a. Eggs of Necator americans

b. Eggs of Enterobius vermicularis

c. Eggs of Hymenolepis nana

d. Eggs of Taeniasolium

Solution. D
Unfertilized eggs of Ascaris, Larva of Strongyloides, Taenia eggs and Operculated eggs of trematodes DO NOT float in a saturated salt
solution.

Correct Answer. d

(109). A 30 year old male patient presents with urethritis. All of thefollowing can be the causative agent except:

a. Neisseria gonorrhoeae

b. Chlamydia trachomatis

c. Trichomonas vaginalis

d. Haemophilus ducreyi

Solution. D
Haemophilus ducreyi can cause genital ulcer but not urethritis.

Correct Answer. d

(110). A veterinary doctor had pyrexia of unknown origin. Hisblood culture in special laboratory media was positive forgram-negative short
bacilli which was oxidase positive.Which one of the following is the likely organism grown inculture?

a. Pasteurella spp

b. Francisella spp

c. Bartonella spp

d. Brucella spp

Copyright © 2014 Delhi Academy of Medical Sciences, All Rights Reserved. 37/116
Solution. D
Points in favour of Brucellosis:
–– Veterinary doctor- Occupational exposure to infected animals.
–– Organism could grow only in special laboratory media.
–– Oxidase positive Gram-negative short bacilli.
–– Brucellosis is one of the common cause of PUO.

Correct Answer. d

(111). A young male patient presented with UTI, on urineexamination pus cells were found but no organisms. Whichmethod would be best used
for culture:

a. McCoy cell line

b. Thayer martin medium

c. LJ. medium

d. Levinthal medium

Solution. A
Probable causes for non gonococcal urethritis (UTI with only pus cells and no bacteria on wet mount examination of urine):
–– Chlamydia trachomatis- commonest agent.
–– Ureaplasma urealyticum & Mycoplasma hominis.
–– Mycobacterium tuberculosis.
–– Herpes virus and Cytomegalovirus.
–– Trichomonas vaginalis and Candida albicans.
Chlamydia trachomatis can be successfully cultivated in McCoy and HeLa cells treated with cycloheximide and grown inmonolayer on
cover slip in shell vials.

Correct Answer. a

(112). Beta lactamase is produced by:

a. E.coli

b. Gonococcus

c. Staphylococcus aureus

d. All of the above

Solution. D
Beta lactamase enzymes are plasmid coded, produced by both gram positive and gram negative organisms.

Correct Answer. d

(113). Which are example of agglutination test:

a. Widal test

b. VDRL test

c. Kahn test

d. Ascoli’s test

Solution. A
Widal test is an agglutination test.
VDRL- Slide flocculation test (Precipitation test).
Kahn test- Tube flocculation test (Precipitation test).
Ascoli’s thermo precipitation test- Ring precipitation test done for Anthrax antigen detection.

Correct Answer. a

(114). Which of the following is associated with class specificantigenic determinants of an immunoglobulin?

a. L:chain

b. H:chain

Copyright © 2014 Delhi Academy of Medical Sciences, All Rights Reserved. 38/116
c. J:chain

d. Variable region

Solution. B
Immunoglobulin is classified to various classes like IgG/ IgA/ IgM/ IgD/ IgD based on the differences betweenconstant regions of Heavy
chain.

Correct Answer. b

(115). A super antigen is a bacterial product that:

a. Binds to B7 and CD28 co-stimulatory molecules

b. Binds to the beta chain of TCR stimulating T cell activation

c. Binds to the CD4 + molecule causing T cell activation

d. Is presented by macrophages to a larger – than: normalnumber of T helper CD4 + lymphocytes

Solution. B
Superantigens:
Act on Vβ region of TCR (T cell receptor).
Dont require antigen presentation by macrophage, directly stimulate non specificT cells.
Leads to massive release of cytokines & polyclonal B cell activation.
Example:
• Staphyloccal toxin- TSST, Exfoliative toxin, Enterotoxins.
• Streptococcal toxin- Str pyrogenic exotoxin A, B, C
• Mycoplasma arthritidis
• Yersinia enterocolitica.
• M.tuberculosis

Correct Answer. b

(116). Fetus of weight 1000 g, crown rump length 23 cm, meconium in intestine, eye lid open and pupillary membrane disappear. Age of fetus

a. 5 months

b. 6 months

c. 7 months

d. 8 months

Copyright © 2014 Delhi Academy of Medical Sciences, All Rights Reserved. 39/116
Solution. (c) 7 months
Ref: Read the text below
Sol:
Assessment of Age in Fetus:
At the end of First Month:
Length is about 1 cm
Weight is about 2.5 gm
At the end of Second Month:
Length is about 4 cms
Weight is about 10 gm
At the end of Third Month:
Length is about 9 cms
Weight is about 30 gm
Eyes Closed and Pupillary Membrane appears
At the end of Fourth Month:
Length is about 16 cms
Weight is about 120 gm
Lanugo hair is seen over the body
Meconium is found in the duodenum
At the end of Fifth Month:
Length is about 25 cm
Weight is about 400 gm
Meconeum is seen at the beginning of large intestine
At the end of Sixth Month:
Length is about 30 cm
Weight is about 700 gm
The testes are seen close to the Kidney
At the end of Seventh Month:
Length is about 35 cm
Weight is about 900-1200 gm
Crown-rump length is about 23 cms
Eyelids open and pupillary membrane disappears
Meconium is found in the entire large intestine
Testes are found at the external inguinal canal
At the end of Eight Month:
Length is about 40 cm
Weight is about 1 Kg to 2.5 Kg
Left testis is present in the scrotum
At the end of Ninth Month:
Length is about 45 cm
Weight is about 2.5 Kg to 3 Kg
Meconeum is seen at the end of large intestine

Correct Answer. c

(117). A 28 year old man consumes an unknown substance and was brought to emergency department. He was unconsciousness and kerosene
odor was coming from his mouth. His BP was 80/60 mm/Hg and pulse rate 110/min.Which of the following will not be done in emergency
management in emergency room?

a. Gastric lavage

b. Atropine

c. Vaso-pressure

d. Maintain airways

Solution. (a) Gastric lavage


Ref: Read the text below
Sol: Kerosene poisoning: Management
- Gastric evacuation for pure petroleum distillate or turpentine ingestion is not recommended.
- Maintain airways
- Corticosteroids
- Absorption of ingested kerosene can be slowed by giving 250 ml of liquid paraffin orally.

Correct Answer. a

(118). A boy of 10 years will have ..……deciduous and …… permanent teeth.

a. 16, 4

b. 12, 8

c. 20, 0

Copyright © 2014 Delhi Academy of Medical Sciences, All Rights Reserved. 40/116
d. 20,4

Solution. (d) 20,4


Ref: Read the text below
Sol:
- (age-5) x 4 = no of permanent teeth in mixed dentition.
- Rest will be the temporary teeth.

Correct Answer. d

(119). Bertillon system is method of identification using

a. Dental record.

b. Finger prints.

c. Anthropometry

d. DNA fingerprinting

Solution. (c) Anthropometry


Ref: Read the text below
Sol:
- The Bertillon system was introduced in the U.S. in 1887 by R.W. McClaughry, Warden of the Illinois State Penitentiary at Joliet.
McClaughry translated Bertillon’s 1885 edition of Signaletic Instructions Including the Theory and Practice of Anthropometrical
Identification from French to English, and its use in the States became quickly and widely accepted. The Bertillon system continued as the
dominant criminal identification method both in the U.S. and Europe for almost three decades.
- In 1903, the case of the “West Brothers” demonstrated the reliability of the emerging science of fingerprint identification over that of
the Bertillon system.

Correct Answer. c

(120). Section of IPC for punishment to medical negligence infecting a normal healthy person with HIV positive needle

a. 166 B

b. 202

c. 203

d. 269

Solution. (d) 269


Ref: Read the text below
Sol:
- Negligent act likely to spread infection of disease dangerous to life.—Whoever unlawfully or negligently does any act which is, and which
he knows or has reason to believe to be, likely to spread the infection of any disease dangerous to life, shall be punished with
imprisonment of either description for a term which may extend to six months, or with fine, or with both.

Correct Answer. d

(121). Pigmentation seen in dead body is due to.

a. Reduced hemoglobin

b. Sulf-methemoglobin

c. Methemoglobin

Copyright © 2014 Delhi Academy of Medical Sciences, All Rights Reserved. 41/116
d. Hematin crystal

Solution. (b) Sulf-methemoglobin


Ref: Read the text below
Sol: Putrefaction:
- First external sign of putrefaction in a body lying in air is usually a greenish discoloration of the skin over the region of cecum.
- Color results from the conversion of hemoglobin of blood into sulf-methemogloboin by hydrogen sulfide formed in large intestine and
escaping into the surrounding tissue. The color appears in 12 to 18 hours in summer and in one to two days in winter.
- Green coloration then spreads over the entire abdomen, external genitals and then patches appear successively on the chest, neck, face,
arms and legs.
- The putrefactive bacteria spread most easily in fluid and tend to colonize the venous system. The superficial veins especially over roots
of limb, thighs, side of abdomen, shoulder, chest and neck are stained greenish brown or purplish red (Linear branching pattern) due to
the hemolysis of RBCs in blood vessels, which stains the wall of the vessel and infiltrates into the tissue, giving a marble appearance. This
occurs soon, but is prominent in 36 to 48 hours. Important point: Intensity of color in post mortem hypostasis depends upon the amount of
reduced hemoglobin in blood.

Correct Answer. b

(122). Surgeon can perform an emergency surgery when the patient is not in a condition to give consent under

a. Sec 91 IPC

b. Sec 92 IPC

c. Sec 87 IPC

d. Sec 89 IPC

Solution. (b) Sec 92 IPC


Ref: Read the text below
Sol: Sec 92 IPC:
- Act done in good faith for benefit of a person without consent
- Nothing is an offence by reason of any harm which it may cause to a person for whose benefit it is done in good faith, even without that
person’s consent, if the circumstances are such that it is impossible for that person to signify consent, or if that person is incapable of
giving consent, and has no guardian or other person in lawful charge of him from whom it is possible to obtain consent in time for the
thing to be done with benefit

Correct Answer. b

(123). Unidentified person from roadside is brought by police to emergency room with disorientation, vitals stable and bilateral pin point pupil.
Diagnosis is

a. Pontine hemorrhage

b. Opioid poisoning

c. Dhatura

d. Barbiturate poisoning

Solution. (b) Opioid poisoning


Ref: Read the text below
Sol:
Opioid poisoning:
- Moist perspiring skin
- Slow pulse
- Triad of coma, pinpoints immobile pupils and respiratory depression

Correct Answer. b

(124). All of the following are method to identify airway involvement except:

a. Hoarseness & stridor

b. ngeing of eyebrows and hair

c. Yellow colored sputum

d. Blistering or edema of the oro-pharynx

Copyright © 2014 Delhi Academy of Medical Sciences, All Rights Reserved. 42/116
Solution. (c) Yellow colored sputum
Ref: Read the text below
Sol:
Assessment of airways in burn patient:
- Singed nasal and facial hairs
- Dysphonia and stridor
- Blistering or edema of the oro-pharynx
- Carbonaceous sputum
- Yellow colored sputum suggests an airway infection

Correct Answer. c

(125). Patient comes with nausea, vomiting and ascending paralysis. Diagnosis of poisoning is

a. Opium

b. Dhatura

c. Conium maculatum

d. Strychnos nux-vomica

Solution. (c) Conium maculatum


Ref: Read the text below
Sol:
Conium maculatum (Hemlock):
- The plant contains coniine alkaloids highest in the unripe fruit and in seed, in leaves especially at flowering time. Clinical presentations:
- Nausea, vomiting and diarrhea
- Mental confusion, Tremor, ataxia and ascending progressive motor paralysis
- Coma and death due to respiratory failure

Correct Answer. c

(126). A smoker states that he has been smoking for 6 years. In the first year he was taking up to 5 sticks per day only. In the next 3 years he
increased it to half pack per day (which was an odd number). In the 5th year, his habits worsened to 1 pack per day. In the last year he
stated that his daily sticks consumption is 2 packs per day. Select the correct statement for mean, median and mode of number of sticks
are

a. 16,10,15

b. 16,10,10

c. 24,15,15

d. 16,16,15

Copyright © 2014 Delhi Academy of Medical Sciences, All Rights Reserved. 43/116
Solution. C

Explanation (contd.)
• Then how would you have solve it: Conceptual; mode & median are same; which is possible in 2 option
A. 16,10,15
B. 16,10,10
C. 24,15,15
D. 16,16,15
• But in the question it is mentioned that the person smokes odd number of cigarettes so answer is C
Explanation (contd.)

Correct Answer. c

(127). Throat swab was done on a child & stool sample was also collected after the child had developed weakness in limbs. Child had no history of any
recent vaccination. Sewer sample were also positive for the virus. No wild strain was found in the community. Strain is:

a. Type 1

b. Type 2

c. Type 3

d. Non typable strain

Copyright © 2014 Delhi Academy of Medical Sciences, All Rights Reserved. 44/116
Solution. B
• This is a case of cVDPV of polio (no vaccination; no wild strain) and this is due Type 2 strain
Explanation (contd.)B

Correct Answer. b

(128). A study was conducted to assess relationship between height and osteoporosis. In this two participants were chosen with the following details. This
is:

a. Matched

b. Overmatched

c. Unmatched

d. Randomized

Copyright © 2014 Delhi Academy of Medical Sciences, All Rights Reserved. 45/116
Solution. B
Matching
• Process of selecting controls so that they are similar to cases with regard to certain variables
• Don’t match for the variable in interest as won’t be able to get statistical results
• Eliminates confounding
• Done in: Case control >> Cohort
Matching Example
• Research question: Assessment of height of a girl and post 30 years osteoporosis

Matching Example (contd.)


• Research question: Assessment of height of a girl and post 30 years osteoporosis

Correct Answer. b

(129). Which method is used to diagnose TB in HIV positive patients?

a. CXR

b. Smear examination

c. Both

Copyright © 2014 Delhi Academy of Medical Sciences, All Rights Reserved. 46/116
d. CBNAAT

Solution. D
• As per the new revised guidelines of RNTCP any patient with HIV positive status; the test to be performed is CBNAAT

Correct Answer. d

(130). Which study design does this diagram pertain to?

a. Case control

b. Cohort

c. Ecological

d. RCT

Solution. C
• The diagram provides country level estimates so pertains to ecological study

Correct Answer. c

Copyright © 2014 Delhi Academy of Medical Sciences, All Rights Reserved. 47/116
(131). Which kind of epidemic curve will this diagram depict?

a. Common source point

b. Common source propagated

c. Mixed

d. Recurrent

Solution. A

Correct Answer. a

Copyright © 2014 Delhi Academy of Medical Sciences, All Rights Reserved. 48/116
(132). What is the median survival in this diagram?

a. 8 years

b. 10 years

c. 12 years

d. 6 years

Solution. B

Correct Answer. b

Copyright © 2014 Delhi Academy of Medical Sciences, All Rights Reserved. 49/116
(133). This diagram is the pictorial representation of which type of study design?

a. RCT

b. Cross sectional

c. Ecological

d. Any of these

Solution. A

Correct Answer. a

Copyright © 2014 Delhi Academy of Medical Sciences, All Rights Reserved. 50/116
(134). In the cycle provided; which is correct?

a. Monitoring done before evaluation

b. Monitoring is done after evaluation

c. Both done simultaneously

d. Not done in this cycle at all

Solution. A

Correct Answer. a

Copyright © 2014 Delhi Academy of Medical Sciences, All Rights Reserved. 51/116
(135). Which cut off would you prefer to take?

a. 3

b. 6

c. 15

d. 1.1

Solution. C
• This is a ROC/ Receiver Operator Curve which is used to select the cut offs
• X- axis: 1 – Specificity/ FP
• Y- axis: Sensitivity/ TP
• Types:
• Straight line at 45o (a): no benefit by this cut off
• (b) & (c): fair & good results
• (d): perfect ROC
• Excellent
• 100% sensitivity & specificity

Correct Answer. c

Copyright © 2014 Delhi Academy of Medical Sciences, All Rights Reserved. 52/116
(136). A person died in emergency. The anaesthetist was resuscitating the patient but in middle his mask fell off and had contact with patient’s
mucosal secretion as patient had coughed out gastric contents. The anaesthetist died after 10 days. No unnatural reasons were found. On
autopsy intra cytoplasmic bodies were found. The most probable infectious agent is:

a. Rabies

b. Botulism

c. Anthrax

d. Cause Unknown

Solution. A
• The picture is that of intra cytoplasmic bodies classically seen in rabies.
• Clinical history shows death in ten days which is a feature of rabies
• Also there is might have been mucosal contact

Correct Answer. a

(137). The intraocular pressure(IOP) was measured in apopulation of 400 people above age of 65. Themean IOP was 25 & SD was 10. What is
the range that would contain IOP of 95% of population:

a. 23-27

b. 21-29

c. 24-26

d. 22-28

Solution. C
• N = 400; Mean = 25; SD = 10; 95% CI = ?????
• SE = SD/ square root of N = 10/ square root 400 = 10/ 20 = 0.5
• CI = Mean ± 2 SE = 25 ± 2 * 0.5 = 24- 26
• CAUTION: Don’t forget to calculate SE whenever you calculate CI

Correct Answer. c

(138). The diagram about correlation shows:

a. Positive

b. Negative

Copyright © 2014 Delhi Academy of Medical Sciences, All Rights Reserved. 53/116
c. No

d. Spurious

Solution. C

Correct Answer. c

(139). In acentre number of nodes dissected during20modified radical mastectomy were plotted in form of curve& there are 3 markings on
curve below. Basedon the distribution, which of following is correct?

a. Mean>median>mode

b. Mode> median> mean

c. Median>mode>mean

d. Mode >mean>median

Copyright © 2014 Delhi Academy of Medical Sciences, All Rights Reserved. 54/116
Solution. A

Correct Answer. a

(140). Match the following and what are the options?:

a. 1 – III; 2 – II; 3 – V; 4 – IV; 5 - I

b. 1 – I; 2 – II; 3- III; 4 – IV; 5 – V

c. 1 – III; 2 – V; 3 – II; 4 – I; 5 – IV

d. 1 – I; 2 – III; 3 – II; 4 – V; 5 - IV

Copyright © 2014 Delhi Academy of Medical Sciences, All Rights Reserved. 55/116
Solution. A

Correct Answer. a

(141). Which is not true for ‘Reduvid Bug’?

a. Also called kissing bug

b. Also called cone nose bug

c. Causes sleeping sickness

d. Causes African Trypnosomiasis

Solution. D
• Tse- tse fly (Glossina palpalis): Sleeping sickness (African trypanosomiasis)
• Reduvid bug (Triatominae/ Kissing/ Assassin/ Cone nose bug): Sleeping sickness (American trypanosomiasis (Chagas Disease)

Correct Answer. d

(142). Which of the following is false about the best indicators for disease burden?

a. Mortality indicator – Proportional Mortality Rate

b. Morbidity indicator – Prevalence

c. Health index – DALY

d. Health index - HALE

Solution. C
Best Indicator of Disease Burden

Correct Answer. c

(143). Which is not correct about the nodal heads for disaster management?

a. Head – Prime Minister

b. Level – CHC

c. Agency – National Disaster Management Authority

Copyright © 2014 Delhi Academy of Medical Sciences, All Rights Reserved. 56/116
d. Ministry - Home

Solution. B
‘Nodal’ in Disaster Management

Correct Answer. b

(144). For which vaccine is the open vial policy applicable?

a. Rota

b. JE

c. Yellow fever

d. IPV

Solution. D
Open Vial Policy
• Reconstituted vaccines (B.C.G, MEASLES, J.E. & ROTAVIRUS) can’t be reused
• All vaccines which are not reconstituted can be reused for up to a maximum of 4 WEEKS/ 28 days if
• Provided that all of the following conditions are met:
• The expiry date has not passed; The vaccines are stored under appropriate cold chain conditions
• The VVM has not been submerged in water
• Aseptic technique has been used to withdraw all doses
• VVM if attached, has not reached the discard point

Correct Answer. d

(145). Which is the highest level of learning?

a. Psychomotor

b. Cognitive

c. Affective

d. All are equal

Solution. A
Types of learning

Correct Answer. a

(146). Which of the following is an advantage of active surveillance?

a. Requires less project staff

b. Is relatively cheaper

c. More accurate

d. Reporting systems can be developed quickly

Copyright © 2014 Delhi Academy of Medical Sciences, All Rights Reserved. 57/116
Solution. C

Correct Answer. c

(147). A 7 day old child at a PHC was diagnosed as jaundice involving palms & soles what is next line of management as per RMNCHA?

a. Reassure mother

b. Refer to SNCU

c. Refer to NBSU

d. Refer to NBCC

Solution. C

Correct Answer. c

(148). Under PMSMY which colour coding is wrong?

a. Green – Normal pregnancy

b. Red – High risk pregnancy

c. Blue – PIH

d. Orange - Systemic diseases

Solution. D
• Pradhan Mantri Surakshit Matritwa Yojana (PMSMY)
• On the 9th of every month, free health check ups for pregnant women in 14 states
• If any poor pregnant women comes to private practitioners then, they should give her free treatment
• Launch date: 09th June 2016
• Under MOHFW
• Colour coded:
• Red: High risk Pregnancy
• Blue: PIH
• Yellow: Pregnancy with co morbidities (DM; STIs; Hypothryroid)
• Green: No risk factor

Correct Answer. d

Copyright © 2014 Delhi Academy of Medical Sciences, All Rights Reserved. 58/116
(149). Why is Vision 2020 named so?

a. Centres to be developed in multiples of 20’s

b. Objective is to achieve blindness to 0.3% by 2020

c. Incentive for every cataract surgery is Rs. 2020/-

d. Ophthalmologists will be made 2020 per state in the country

Solution. B
• Objective of Vision 2020 is to reduce the prevalence of blindness to 0.3% Vision 2020: The Right to Sight
• Global initiative to reduce avoidable (preventable & curable) blindness by 2020
• Targeted Diseases for Vision 2020:
• World:
• Cataract
• Refractive error & low vision
• Trachoma
• Childhood blindness
• River Blindness
• India: World +
• Glaucoma
• Diabetic Retinopathy
• Corneal blindness
• No river blindness
• Ophthalmologist is present at all levels of health care except Vision where we have a optometrist Vision 2020

Correct Answer. b

(150). Which is the following is not correct for prevention of cervical cancer?

a. Pap smear is secondary prevention

b. Cervical cancer vaccine is primary prevention

c. No sexual intercourse form of primordial prevention

d. Pap smear is primary prevention

Solution. D
• Prevention: Ask only two questions to yourself in sequence to get answers for preventive strategies
• Q.1. What it prevents?
• Primordial: Risk factor
• Primary: Disease
• Secondary: Complication
• Tertiary: Further Complication
• Quaternary: Over Diagnosis
Explanation (contd.)
• Q.2. At what stage it prevents?
• Pre pathogenesis: Primordial & primary
• Pathogenesis: Secondary & Tertiary
• Strategies:
• Primordial: Health Promotion
• Primary: Health Promotion & Specific Protection
• Secondary: Screening for disease (Screening for risk factor is primary)
• Tertiary: Disability Limitation & Rehabilitation
If you have any queries
• Facebook Club
• Or mail at sidykp@gmail.com

Correct Answer. d

Copyright © 2014 Delhi Academy of Medical Sciences, All Rights Reserved. 59/116
(151). All of the following the clinical findings are consistent with severe mitral stenosis except

a. Atrial fibrillation

b. Pulsatile liver

c. Opening snap late after S2

d. Pulmonary vascular congestion

Solution. (c) Opening snap late after S2


Ref: OP Ghai- 387
Sol:
- “Severity of mitral stenosis is clinically judged by the distance between the opening snap and the aortic component of S2.
The closer the opening snap to S2, the more severe is the obstruction

Correct Answer. c

(152). Regarding cryptic miliary tuberculosis all are correct except

a. It has a chronic course

b. It causes mild intermittent fever and anemia

c. Meningeal involvement precedes death

d. It is commonly seen in children

Solution. (d) It is commonly seen in children


Ref: Read the text below
Sol:
- Miliary TB is due to hematogenous spread of tubercle bacilli.
- The lesions are usually yellowish granulomas 1–2 mm in diameter that resemble millet seeds. - - -Clinical manifestations are nonspecific
and protean, depending on the predominant site of involvement.
- Fever, night sweats, anorexia, weakness, and weight loss are presenting symptoms in the majority of cases.
- Physical findings include hepatomegaly, splenomegaly, and lymphadenopathy.
- Eye examination may reveal choroidal tubercles, which are pathognomonic of miliary TB, in up to 30% of cases.
- Frequently, chest radiography reveals a miliary reticulonodular pattern (more easily seen on underpenetrated film), although no
radiographic abnormality may be evident early in the course and among HIV-infected patients.
- Other radiologic findings include large infiltrates, interstitial infiltrates (especially in HIV-infected patients), and pleural effusion.
- Sputum-smear microscopy is negative in most cases.
- The TST may be negative in up to half of cases, but reactivity may be restored during chemotherapy.
Cryptic miliary TB
- A rare presentation seen in the elderly, chronic course
- characterized by mild intermittent fever, anemia& ultimately meningeal involvement preceding death.
- Nonreactive miliary TB
- An acute septicemic, rapidly fatal form, due to massive hematogenous dissemination of tubercle bacilli.
- Pancytopenia is common & at postmortem examination, multiple necrotic but nongranulomatous (“nonreactive”) lesions are detected.

Correct Answer. d

Copyright © 2014 Delhi Academy of Medical Sciences, All Rights Reserved. 60/116
(153). A 37-year-old woman complains of headache and blurry vision that have been present for a year and are slowly getting worse. As part of
her evaluation an MRI is obtained and shown below:

What is the most likely diagnosis in this patient?

a. Brain abscess

b. Glioblastoma

c. Low-grade astrocytoma

d. Meningioma

Solution. (d) Meningioma


Ref: Read the text below
Sol:
- This figure illustrates a mass attached to the meninges with a dural tail. Other dural tumors may appear this way, but of the options
listed, the meningioma is by far the most likely to appear this way.
- Meningiomas derive from the cells that give rise to the arachnoid granulations. They are usually benign and attached to the dura. They
rarely invade the brain.
- They are more frequent in women than men and have a peak incidence in middle age. Total surgical resection of a meningioma is
curative. Low-grade astrocytoma and high-grade astrocytoma (glioblastoma) often infiltrate into adjacent brain and rarely have the clear
margins seen in this figure.
- Oligodendroma comprise ~15% of all gliomas and show calcification in roughly 30% of cases. They have a more benign course and are
more responsive than other gliomas to cytotoxic therapy.
- For low-grade oligodendromas, the median survival is 7–8 years. Brain abscess will have distinctive ring-enhancing features with a
capsule, often have mass effect, and will have evidence of inflammation on MRI scanning.

Correct Answer. d

(154). A patient with pleural effusion has the following test results
- Serum protein= 7G/dL
- Pleural fluid protein=5G/dL
- Serum glucose=90mg/dL
- Pleural fluid glucose=20mg/dL
- Pleural fluid LDH =300 units
Probable diagnosis is

a. CHF patient not on diuretics

b. CHF patient with diabetes and on insulin

c. Renal failure

d. Rheumatoid arthritis

Solution. (d) Rheumatoid arthritis


Ref: Read the text below
Sol:
- Pleural fluid Glucose < 60 mg/dL seen in effusions due to Malignancy, Bacterial infections and Rheumatoid pleuritis.

Correct Answer. d

Copyright © 2014 Delhi Academy of Medical Sciences, All Rights Reserved. 61/116
(155). Which of the following most often presents with a diarrhea state as its first symptom ?

a. Cancer

b. Hepatitis

c. Cholecystitis

d. HIV syndrome

Solution. (d) HIV syndrome


Ref.: Read the text below
Sol:
- Diarrhea is the most common gastrointestinal symptom of the immunocompromised patient. The initial stages may include a diarrhea
state for 1 to 2 weeks.
- The patient with full-blown AIDS may experience multiple diarrhea states that are severe and recurrent due to the opportunistic nature
of bowel flora.
- Bacteremia and sepsis are often seen and must be treated aggressively.
- The other choices can all cause diarrhea states, but they are seldom the presenting symptom.

Correct Answer. d

(156). The most important hormone to replace in treating complete panhypopituitarism is

a. Cortisol

b. Thyroxine

c. Growth hormone

d. Estrogen or testosterone

Solution. (a) Cortisol


Ref.: Read the text below
Sol:
- Cortisol and thyroxine are both essential for life, but cortisol is more rapidly life threatening when severely deficient.
- When both are deficient, cortisol should be replaced first, because thyroxine increases cortisol metabolism and can therefore worsen
cortisol deficiency.

Correct Answer. a

(157). In patients with severe structural lung disease, such as bronchiectasis the infective agent most likely to cause pneumonia is

a. H. Influenza

b. Pneumococcus

c. Legionella

d. P. aeruginosa

Solution. (d) P. aeruginosa


Ref: Read the text below
Sol:
- Pseudo aeruginosa has propensity for colonising damaged airways & evading host defence mechanism

Correct Answer. d

Copyright © 2014 Delhi Academy of Medical Sciences, All Rights Reserved. 62/116
(158). In a patient with following CXR and CT scan, the most appropriate method for measuring lung volumes is

a. Body plethysmography

b. Diffusing capacity of carbon monoxide

c. Spirometry

d. Helium dilution

Solution. (a) Body plethysmography


Ref: Read the text below
Sol:
- Given CXR and CT chest suggests emphysematous changes
- Spirometry does not measure total lung capacity because itcannot account for residual volume.
- The most frequently used and accurate measures of lungvolumes are steady-state helium dilution lung volumes and body
plethysmography.
- Subsequently, body plethysmography is the preferredmethod for lung volume measurement in these disease states.
- To perform body plethysmography,the patient sits in a sealed box and pants against a closed mouthpiece. Panting results in changes
inthe pressure of the box that, when compared with changes at the mouthpiece, can be used tocalculate lung volumes.
- This method measures total thoracic gas volume and is more accurate thanhelium dilution.

Correct Answer. a

(159). The corrected QT interval (QTc interval) is:

a. 0.12-2 secs

b. > 0.5 secs

c. 2 to 3 secs

< 0.46 secs>d.

Copyright © 2014 Delhi Academy of Medical Sciences, All Rights Reserved. 63/116
Solution. (d) < 0.46 secs
Ref: Harrison’s- 1389
Sol:

Correct Answer. d

(160). Micro albuminuria is defined as

a. 0.3–0.5 g of 24 hrs urine protein

< 0.03 g of 24 hrs protein>b.

c. 0.03–0.3 g of 24 hrs urine protein

d. > 2.5 g 24 hrs urine protein

Solution. (c) 0.03–0.3 g of 24 hrs urine protein


Ref: Davidson’s, - 479
Sol :

Correct Answer. c

(161). DLCO is a very sensitive test for interstitial lung diseases, in which it is decreased. Which among the following conditions cause an
increase in DLCO?

a. Pulmonary fibrosi

b. Pulmonary artery hypertension

c. Alveolar hemorrhage

d. ILD

Solution. (c) Alveolar hemorrhage


Ref: Read the text below
Sol:
DLCO: Diffusion capacity (or Transfer factor) of lung Carbon monoxide
- Normal to elevated in asthma
- Reduced in all parenchymal restrictive lung diseases and pulmonary HT
- Increased in alveolar hemorrhage like Goodpasture syndrome

Copyright © 2014 Delhi Academy of Medical Sciences, All Rights Reserved. 64/116
Correct Answer. c

(162). A 40 year old male c/o of diarrhea for many years has stool osmotic gap of 200 and also gives a history of decrease in stool volume during
fasting. The most likely etiology is

a. Chronic Shigella Infection

b. Lactase deficiency

c. Biscodyl use

d. Gastrinoma

Solution. (b) Lactase deficiency


Ref: Read the text below
Sol:
- Stool osmotic gap of more than 150 suggests osmotic diarrhea.The causes include osmotic laxative, lactase deficiency and lactulose.
- Chronic alcohol ingestion by damaging enterocytes leads to secretory diarrhea.
- Biscodyl is stimulant laxative casing secretory diarrhrea.Gastrinoma causes both secretory diarrhea as well as steatorrhea.

Correct Answer. b

(163). A patient with steatorrhea has microcytic anemia.The urine excretion of DXylose after 25 gm oral D-Xylose is 2.5 gm.He is suffering from

a. Celiac sprue

b. Chronic pancreatitis

c. Tropical sprue

d. Ileal resection

Solution. (a) Celiac sprue


Ref: Read the text below
Sol:
- Malabsorption syndrome affecting proximal intestine will lead to iron,folic acid and calcium deficiency as these nutrients are absorbed
from duodenum and jejunum.
- D xylose is also absorbed from proximal mucosa hence will not be absorbed in conditions affecting proximal mucosa.
- Celiac sprue predominantly affects proximal intestine.Tropical sprue maily affects distal intestine.
- Chronic pancreatitis will not affect mocusa of intestine therefore D xylose test will be normal.

Correct Answer. a

(164). The pathologic specimen given below shows the only intracranial lesion found in this patient. This patient would be expected to have
exhibited which of the following
symptoms?

a. Seizures

b. Gait ataxia

c. Hemiparesis

d. Visual loss

Copyright © 2014 Delhi Academy of Medical Sciences, All Rights Reserved. 65/116
Solution. (b) Gait ataxia
Ref: Read the text below
Sol:
- This specimen is a transverse section through the brainstem and cerebellum.
- There is a large area of discoloration and disturbed anatomy in the left cerebellar hemisphere that is producing little mass effect.
- Because this is the only lesion postulated for this patient, there is no reason to suspect seizure activity, because that phenomenon would
be unlikely in the absence of a cerebrocortical (or at least cerebral) lesion.
- The other findings listed would similarly not be expected in a patient with cerebellar damage.

Correct Answer. b

(165). Amyloidosis of kidney may be seen in the following except :

a. Enteric fever

b. Ulcerative colitis

c. Suppurative lung disease

d. Hansen’s disease

Solution. (a) Enteric fever


Ref.: Robbins - 258
Sol:
Seondary (reactive) systemic amyloidosis:
- Protein type – AA type
- Organs commonly involved – liver, spleen, kidney, adrenals,
- Occurs in – chronic infections (Suppurative), tuberculosis, bronchiectasis, chronic osteomyelitis, chronic pyelonephritis, leprosy,
autoimmune disorders (RA, dermatomyositis, scleroderma) inflammatory bowel disease (ulcerative colitis and Crohn’s disease), some
tumours (renal cell carcinoma, Hodgkin’s disease).

Correct Answer. a

(166). Acute symptomatic sinus bradycardia usually responds to :

a. Adrenalin

b. Dopamine

c. Atropine

d. Norepinephrine

Solution. (c) Atropine


Ref.: Robbins Pathologic Basis of Disease, - 1416
Sol:
- Sinus bradycardia – A sinus rate less than 60 per minute may occur in normal people during sleep and is a common finding in athelets.
- Some pathological causes – MI, sick sinus syndrome, hypothermia, hypothyroidism, cholestatic jaundice, raised intracranial pressure,
drugs (β-adrenoceptor antagonist, digoxin, verapamil).
- Acute symptomatic sinus bradycardia usually responds to IV atropine 0.6 mg.

Correct Answer. c

(167). Most common cause of refractory hypertension is?

a. Noncompliance of patient

b. Obstructive sleep apnea

c. Pheochromocytoma

d. Renovascular disease

Solution. (a)Non compliance of patient


Ref.: Read the text below
Sol :
- The most common causes of refractory hypertension is non compliance other causes are obstructive sleep apnea, hyperaldosteronism,
renal parenchymal disease and renal artery stenosis.
- Hyperaldosteronism has been found in as many as 20 percent of patients with resistant hypertension.

Correct Answer. a

Copyright © 2014 Delhi Academy of Medical Sciences, All Rights Reserved. 66/116
(168). Which of the following is a Sino atrial disease?

a. Atrial ectopic

b. Ventricular ectopic

c. Sinus arrest

d. All of the above

Solution. (c) Sinus arrest


Ref.: Read the text below
Sol:
- SA node dysfunction may be completely asymptomatic and manifest as an ECG anomaly such as sinus bradycardia; sinus arrest and exit
block; alternating supraventricular tachycardia, usually atrial fibrillation, and bradycardia.

Correct Answer. c

(169). The curve depicted below plots the normal relationship of arterial PO2 and percentage of hemoglobin saturation with other variables
controlled at pH 7.4, PaCO2 40 kPa, temperature 37C (98.6F), and hemoglobin 15 g/dL. Which of the following statements regarding this
oxygen dissociation relationship is true?

a. Modest decrements of arterial PO2 have a major effect on alveolar oxygen uptake

b. Modest decrements of hemoglobin saturation have a major effect on tissue oxygen uptake

c. The curve shifts to the left with acidosis

d. The curve shifts to the left following banked blood transfusion

Solution. (d) The curve shifts to the left following banked blood transfusion
Ref: Read the text below
Sol:
- The shape of the oxygen dissociation curve translates into several physiologic advantages.
- The relatively flat slope above a PO2 of 50 pKa means that, in this region of the curve, hemoglobin saturation decreases slightly with
decrements in PO2; loading of oxygen at the alveolar level is therefore affected minimally with mild to moderate degrees of hypoxemia.
- The steeper slope at the lower end of the curve means that, as the hemoglobin becomes desaturated, arterial PO2 drops only minimally,
and a gradient that favors oxygen diffusion into tissue cells is maintained.
- Acidosis, a rise in PaCO2, and elevation of temperature all shift the curve to the right, which enhances tissue oxygen uptake. Red blood
cell organic phosphates, particularly 2,3-diphosphoglycerate (2,3-DPG), also affect the dissociation curve. Banked blood, being low in 2,3-
DPG, shifts the curve to the left and therefore decreases tissue oxygen uptake. 2,3-DPG levels increase with chronic hypoxia.
- Chronic lung disease, therefore, results in a shift of the curve to the right, which enhances oxygen delivery to peripheral tissues.

Correct Answer. d

(170 Which one of the following is associated with shift of potassium into
). cells?

a. Calcium gluconate administration

b. Intravenous furosemide

c. Dialysis

d. Salbutamol inhalation

Copyright © 2014 Delhi Academy of Medical Sciences, All Rights Reserved. 67/116
Solution. (d) Salbutamol inhalation
Ref.: CMDT -899
Sol:

Correct Answer. d

(171). Translocation of bcr-able gene is characteristically seen in :

a. Chronic myeloid leukemia

b. Acute myeloid leukemia

c. Chronic lymphatic leukemia

d. Acute lymphatic leukemia

Solution. (a) Chronic myeloid leukemia


Ref.: Read the text below
Sol:
- CML is characterized by specific chromosomal abnormality, Philadelphia chromosome, a reciprocal translocation between long arm of
chromosome 9 and 22.
- The portion of 9q that is translocated contains abl a proto – oncogene.
- The able gene receives by specific site on 22q that is bcr. The effusion gene bcr/abl produces a novel gene and it differs from normal
transcript of the able gene in that it possesses tyrosine kinase.
- Evidence of bcr/able gene is patholgenic of leukemia.

Correct Answer. a

(172). Which one of the following statements is true of stoke – Adams attack?

a. It is usually preceded by an aura

b. Focal neurological signs are commonly observed during the attack

c. Caused by high degree of atrioventricular block

d. Caused by recurrent paroxysmal tachyarrhythmias

Solution. (c) It is usually caused by high degree of atrioventricular block


Ref.: Read the text below
Sol:
- Adams – stoke attacks – Episodes of ventricular asystole may complicate complete heart block or Mobitz type II second degree AV block
and can occur in patients with sinoatrial disease.
- This may cause recurrent syncope or Adams – stoke attacks.
- Episode is characterized by – sudden loss of consciousness without warning.
- Convulsion due to cerebral ischemia in prolonged asystole. In contrast to epilepsy, recovery is rapid.

Correct Answer. c

(173). Which one of the following is an absolute contraindication to the use of thrombolytic agent in the setting of an acute anterior wall
myocardial infarction?

a. History of CVA with hemiparesis one month ago

b. Diabetic retinopathy

Copyright © 2014 Delhi Academy of Medical Sciences, All Rights Reserved. 68/116
c. Patient’s age more than 70 years

d. Patient is on warfarin for AF with INR ration 1.8

Solution. (a) History of CVA with hemiparesis one month ago


Ref.: Read the text below
Sol:
- Contraindication of fibrinolytic agents:
- Clear contraindication –
(i) History of cerebrovascular hemorrhage at any time,
(ii) Nonhemorrhagic stroke or other cerebrovascular event within the past year.
(iii) Marked hypertension (systolic > 180 mmHg and/or >110 mmHg) at any time during acute presentation
(iv) Suspicion of aortic dissection
(v) Active internal bleeding (excluding menses).
- Relative contraindications – (i) current use of anticoagulants (INR > 2), (ii) A recent (< 2 wks) invasive or surgical procedure, (iii)
prolonged (> 10 min) CPR, (iv) known bleeding diathesis, (v) pregnancy, (vi) hemorrhagic ophthalmic conditions (e.g. hemorrhagic
diabetic diabetic retinopathy), (vii) active peptic ulcer, (viii) history of severe hypertension that is currently adequately controlled.

Correct Answer. a

(174). In tropical splenomegaly syndrome, there is increased serum level of:

a. IgA

b. IgE

c. IgG

d. IgM

Solution. (d) IgM


Ref.: Harrison’s - 223
Sol:
- Tropical splenomegaly syndrome (hyperactive malarial splenomegaly) – An abnormal immune response to repeated infections that is
characterized by massive splenomegaly, hepatomegaly, marked elevations in serum titers of IgM and malarial antibody, hepatic sinusoidal
lymphocytosis.
- This syndrome has been associated with the production of cytotoxic IgM antibodies to CD8+T lymphocytes, antibody to CD5+T
lymphocyte and increase in the ratio of CD4+T cells to CD8+T cells.
- Splenomegaly and anemia usually resolve with proguanil 100 mg daily, which should continued for life to prevent relapse.

Correct Answer. d

(175). Which of the following electrocardiogram findings, occurring shortly after the onset of acute myocardial infarction, is associated with an
increased mortality?

a. Second-degree Mobitz type I atrioventricular block.

b. First-degree atrioventricular block.

c. New right bundle branch block.

d. Sinus bradycardia.

Solution. (c) New right bundle branch block


Ref– Read the text below
Sol:
- New-onset right bundle branch block is most commonly seen in the setting of an anteroseptal myocardial infarction. It portends an
increase in mortality as it often leads to complete heart block.
- First-degree atrioventricular block is usually benign. Sinus bradycardia is not associated with an increased mortality in the absence of
hemodynamic instability and may be protective by reducing myocardial oxygen demand.
- Second-degree Mobitz type I (Wenckebach) is the most common form of second-degree atrioventricular block occurring in the setting of
acute myocardial infarction. It rarely progresses to complete heart block.

Correct Answer. c

(176). Hypocalcemia is characterized by all of the following features, except :

a. Numbness and tingling of circum oral region

b. Hyperactivity of tendon reflexes

c. Shortening of QT interval in ECG

Copyright © 2014 Delhi Academy of Medical Sciences, All Rights Reserved. 69/116
d. Carpopedal spasm

Solution. (c)Shortening of QT interval in ECG


Ref: Read the text below
Sol:
- The QT interval on ECG is prolonged in hypocalcemia and shortened in hypercalcemia.

Correct Answer. c

(177). The syndrome of inappropriate ADH is characterized by the following;

a. Hyponatremia and urine sodium excretion > 20 mEq/L

b. Hypernatremia and urine sodium excretion > 20 mEq/L

c. Hyponatremia and hyperkalemia

d. Hypernatremia and hypokalemia

Solution. (a)Hyponatremia and urine sodium excretion > 20 mEq/L


Ref: Read the text below
Sol:
- SIADH is characterized by hypoosmotic hyponatremia in the setting of inappropriately concentrated urine (urine osmolality greater than
100mosm/Kg).
- Patients are typically normovolemic and have normal Na+ balance. They tend to be mildly volume expanded secondary to water
retention and have a urine excretion rate equal to intake (urine Na+ concentration >40 mmol/L).
- By definition, they have normal renal, adrenal thyroid function and usually have normal K+ and acid base balance.

Correct Answer. a

(178). A 40year old male patient has been having skin lesions over elbow (shown in image) for the past 1 month. His image on the left side is
prior to treatment. He was prescribed an oral drug, the response to which was dramatic and itching reduced significantly (image on
right). He also reports bloating and gaseous distension with intermittent diarrhoea. He is a smoker and also has hyperlipidemia. What is
the most probable diagnosis?

a. Insect bite reaction

b. Psoriasis

c. Dermatitis herpetiformis

d. Eczema

Copyright © 2014 Delhi Academy of Medical Sciences, All Rights Reserved. 70/116
Solution. (c) Dermatitis herpetiformis
Ref:Read the text below
Sol :
Dermatitis herpetiformis (DH) is an inflammatory cutaneous disease with a chronic relapsing course, pruritic papules and vesicles , and
typical histopathological and immunopathological findings. DH usually presents with symmetrical, grouped polymorphic lesions consisting
of erythema, red plaques, and papules, involving the extensor surfaces of the knees, elbows, shoulders, buttocks, sacral region, neck,
face, and scalp. By contrast, herpetiform vesicles, which reflect the name of the disease, may occur later or are often immediately
excoriated, resulting in erosions. The finding of granular IgA deposits along the dermal–epidermal junction or at the papillary tips by
direct immunofluorescence (DIF) assay, together with positive results for anti-tissue transglutaminase antibody testing, allows the
diagnosis. Patients with DH usually show the specific antibodies. Among them, IgA anti-tTG antibodies, IgA anti-endomysium antibodies
(EMAs), IgA and IgG anti-deamidated synthetic gliadin-derived peptides (DGP), and IgA anti-epidermal transglutaminase (eTG) antibodies
are positive. A gluten-free diet should be started in association with drugs, such as oral dapsone, that are able to control the skin
manifestations rapidly.

Correct Answer. c

(179). A 3 months old male infant developed a furuncle for which he was given a course of antibiotics. A few days later, he developed extensive
peeling of the skin with fever. There were no mucosal lesions and the baby was not very toxic. Diagnosis is:

a. Toxic epidermal necrolysis

b. Staphylococcal scalded skin syndrome

c. Steven Johnson syndrome

d. Infantile pemphigus

Solution. (b) Staphylococcal scalded skin syndrome


Ref: Read the text below
Sol:
Staphylococcal scalded skin syndrome.
- It is caused by Staphylococcus aureus of phage group II (here furuncle) which produce epidermolytic toxins.
- These toxins cleave desmoglein 1 and produce widespread peeling of stratum corneum.
- Since desmoglein 1 is not expressed in mucosae, there is no mucosal involvement.
- Steven Johnson syndrome and Toxic epidermal necrolysis are drug reactions with purpuric lesions, severe mucosal involvement and
deeper level of split with full thickness peeling of epidermis.
- In SSSS patient is generally well but in SJS and TEN patient is quite ill.

Correct Answer. b

(180). According to new research, which is most strongly associated with pathogenesis of psoriasis

a. Th1

b. Th2

c. Th17

d. Th18

Solution. (c) Th17


Ref: Read the text below
Sol:
- Psoriasis earlier believed to be mediated by predominantly TH1 cells (CD 8+); but now TH17 appears to be most important pathogenic
subset of T- cells induced by IL-23 from psoriatic plaques.
- IL-8 may be important for attracting neutrophils in the plaques (esp in pustular psoriasis).

Correct Answer. c

(181). “Islands of sparing” are a classical feature of

a. Pityriasis versicolor

b. Pityriasis rosea

c. Pityriasis rubra pilaris

d. Pityriasis alba

Copyright © 2014 Delhi Academy of Medical Sciences, All Rights Reserved. 71/116
Solution. (c) Pityriasis rubra pilaris
Ref: Read the text below
Sol:
- Palmoplantar keratoderma (orange-red, waxy, hyperkeratosis of palms and soles known as PRP sandal)
- Red (salmon colored ) scaly plaques with “islands of sparing” (“nappes claires”) i.e normal skin within erythematous plaques
- Perifollicular, hyperkeratotic, rough, horny papules over trunk and especially over digits simulating a “nutmeg grater” appearance and
feel.

Correct Answer. c

(182). A pregnant female presents with lesions as shown in image.Which of the treatment modalities is recommended?

a. Cryosugery

b. Trichloroacetic acid

c. Imiquimod

d. Podophyllin

Solution. (a) Cryosugery


Ref:Read the text below
Sol :
- Podophyllin is contraindicated in the treatment of Condyloma acuminata (venereal warts) in pregnancy as it is teratogenic. Imiquimod is
also not approved for this use in pregnancy. Cryotherapy is better than TCA here.

Correct Answer. a

(183). Condyloma lata can be treated with which of the following ?

a. Podophyllin

b. Trichloroacetic acid

c. Interferon alpha

d. Penicillin

Solution. (d) Penicillin


Ref: Read the text below
Sol :
- Condyloma lata is a feature of secondary syphilis and hence needs penicillin

Correct Answer. d

(184). A 35 year old woman presents with a history of erythematous papulopustular lesions on the face. She reports diffuse redness on her
cheeks. The most likely diagnosis in the patient is-

a. Acne vulgaris

b. Rosacea

Copyright © 2014 Delhi Academy of Medical Sciences, All Rights Reserved. 72/116
c. Atopic dermatitis

d. Seborrheic dermatitis

Solution. (b) Rosacea


Ref: Read the text below
Sol :
- Rosacea has flushing episodes with telangiectasia (Erythemato-telangiectatic variant). Later due to inflammation, there is
papules/pustules/Rhinophyma
- Acne will have comedones and no redness. Atopic dermatitis has itching and eczematous eruptions while seborrheic dermatitis will have
scaling (yellow greasy) on seborrheic areas with itching

Correct Answer. b

(185). The dressing for a pressure ulcer should

a. Adhere firmly and dry quickly

b. Keep the ulcer moist and the surrounding tissue dry

c. Keep the ulcer tissue dry and the surrounding tissue moist

d. Keep both the ulcer and surrounding tissue moist

Solution. (b) Keep the ulcer moist and the surrounding tissue dry
Ref: Read the text below
Sol :
- The cardinal rule when choosing a dressing is to keep the ulcer tissue moist to promote healing, and the surrounding tissue dry.
- If the surrounding tissue is kept moist this may increase skin breakdown in this area.
- A dressing that adheres to the wound will remove both vital and nonvital tissue, and actually damages the wound.

Correct Answer. b

(186). A 15-year-old boy presents with an as yet unexplained, painless but dramatic swelling of the upper lip since 2 days. The likely diagnosis is

a. Cellulitis

b. Erysipelas

c. Contact dermatitis

d. Angioedema

Solution. (d) Angioedema


Ref: Read the text below
Sol :
- The lip is one of the more common areas involved in angioedema, but many others have been reported, including palms and soles, where
it tends to be slightly tender to touch.
- Cellulitis is commonly misdiagnosed, and is incorrect in this case, since it, unlike angioedema, would be red, hot, and tender, and would
probably not resolve within 48 h, even with treatment.

Correct Answer. d

(187). Which of the following is known to cause hand-foot-and-month disease ?

a. Poxvirus

b. Treponema pallidum

c. Coxsackie virus

d. Human papillomavirus

Solution. (c) Coxsackie virus


Ref: Read the text below
Sol :
- Hand-foot-and-mouth disease is associated with infection by subtype A 16 of the coxsackievirus family.
- Poxvirus causes molluscum contagiosum.
- Treponema pallidum is a spirochete that causes syphilis.
- Human papillomavirus causes condylomata acuminate (genital warts).

Correct Answer. c

Copyright © 2014 Delhi Academy of Medical Sciences, All Rights Reserved. 73/116
(188). All of the following neurotransmitters are involved in pathophysiology of anxiety, except?

a. GABA

b. Norepinephrine

c. Serotonin

d. Glutamate

Solution. D. Glutamate
The three neurotransmitters involved in anxiety are GABA (decrease), Norepinephrine (increase) and serotonin dysregulation.

Correct Answer. c

(189). Who is the father of modern psychiatry?

a. Sigmund Freud

b. Johann Reil

c. Phillipe Pinel

d. Carl Jung

Solution. C. Phillipe Pinel


Phillipe pinel unchained the mental patients and gave a humanistic approach to psychiatry, thats why he is considered the father of
modern psychiatry.
Carl Jung is father of analytical psychology.
Johann Reil coined the term psychiatry.
Sigmund Freud is father of psychoanalysis and dream analysis.

Correct Answer. c

(190). First symptom to disappear with treatment of schizophrenia

a. Apathy

b. Auditory hallucinations

c. Poverty of thoughts

d. Anhedonia

Solution. B. Auditory hallucinations


Hallucination and Delusion are positive symptoms in schizophrenia and anhedonia, apathy are negative symptoms. Positive symptoms
respond better to treatment.

Correct Answer. b

(191). A 75 years old male was operated for fracture neck of femur and since next day develops irritability, disorientation & has been talking to
imaginary people. Diagnosis is

a. Senile dementia

b. Delirium

c. Senile Psychosis

d. Post-operative Psychosis

Solution. B. Delirium
Disorientation, talking to imaginary people (visual hallucinations), acute onset (1 day) after fracture in elderly, points towards diagnosis of
DELIRIUM.

Correct Answer. b

(192). A 6 year old boy has been diagnosed with ADHD and started on Methylphenidate. Which of the following serious side effects should the
child psychiatrist warn the boy’s parents about?

Copyright © 2014 Delhi Academy of Medical Sciences, All Rights Reserved. 74/116
a. Tics

b. Cardiac conduction abnormalities

c. Choreiform Movement

d. Leukopenia

Solution. A. TICS
Methylphenidate increases dopamine and can precipitate tics. It can exacerbate preexisting tic disorder, so in case of ADHD with
comorbid tics, it is avoided and ATOMOXETINE is drug of first choice

Correct Answer. a

(193). Which is false regarding Lewy body dementia?

a. Fluctuating cognitive impairement

b. Neuroleptic sensitivity

c. Recurrent visual hallucinations

d. Cerebellar symptoms

Solution. D. Cerebellar symptoms


Lewy body dementia is characterised by:
• Parkinson like movement problems
• Visual hallucinations
• Fluctualting cognitive impairment
• Sensitivity to neuroleptics
ICD 11 quotes:
"Dementia due to Lewy body disease is the second most common form of dementia in the elderly after Alzheimer disease. The precise
etiology is unknown but involves abnormal alpha-synuclein protein folding and aggregation with Lewy body formation primarily in the
cortex and brainstem. Onset is insidious with attentional and executive functioning deficits typically reported as the initial presenting
complaint. These cognitive deficits are often accompanied by visual hallucinations and symptoms of REM sleep behaviour disorder.
Hallucinations in other sensory modalities, depressive symptoms, and delusions may also be present. The symptom presentation usually
varies significantly over the course of days necessitating longitudinal assessment and differentiation from Delirium. Spontaneous onset of
Parkinsonism within approximately 1 year of the onset of cognitive symptoms is characteristic of the disease."

Correct Answer. d

(194). Double depression means

a. Depression occurring because of two different reasons resulting in double severity

b. Depression occurring twice in a row

c. Depression superimposed on dysthymia

d. Depression superimposed on dementia

Solution. C. Depression superimposed on dysthymia.


Dysthymic disorder is characterized by a persistent depressive mood (i.e., lasting 2 years or more), for most of the day, for more days
than not. In children and adolescents depressed mood can manifest as pervasive irritability. The depressed mood is accompanied by
additional symptoms such as markedly diminished interest or pleasure in activities, reduced concentration and attention or
indecisiveness, low self-worth or excessive or inappropriate guilt, hopelessness about the future, disturbed sleep or increased sleep,
diminished or increased appetite, or low energy or fatigue. (Ref: ICD 11) Double depression is depression superimposed on dysthymia.

Correct Answer. c

(195). All of the following are true about Folie a deux except:

a. Sharing of a delusion between two persons who are closely knit

b. Only one person has authentic delusions due to psychiatric disorder

c. On separation of the two persons, both will be cured of their delusion

d. On separation of the two, dependent individual may give up his delusions

Copyright © 2014 Delhi Academy of Medical Sciences, All Rights Reserved. 75/116
Solution. c. On separation of the two persons, both will be cured of their delusion
Shared psychotic disorder (also referred to over the years as shared paranoid disorder, induced psychotic disorder, folie impose, and
double insanity) was rst described by two French psychiatrists, Lasegue and Falret, in 1877, who named it folie á deux. In DSM-5, this
disorder is referred to as “Delusional Symptoms in Partner of Individual with Delusional Disorder,” an unnecessary nomenclature change
in the view of most psychiatrists. It is probably rare, but incidence and prevalence gures are lacking, and the literature consists almost
entirely of case reports. The disorder is characterized by the transfer of delusions from one person to another. Both persons are closely
associated for a long time and typically live together in relative social isolation. In its most common form, the individual who rst has the
delusion (the primary case) is often chronically ill and typically is the inuential member of a close relationship with a more suggestible
person (the secondary case) who also develops the delusion. The person in the secondary case is frequently less intelligent, more gullible,
more passive, or more lacking in self-esteem than the person in the primary case. If the pair separates, the secondary person may
abandon the delusion, but this outcome is not seen uniformly. The occurrence of the delusion is attributed to the strong inuence of the
more dominant member. Old age, low intelligence, sensory impairment, cerebrovascular disease, and alcohol abuse are among the factors
associated with this peculiar form of psychotic disorder. A genetic predisposition to idiopathic psychoses has also been suggested as a
possible risk factor. Other special forms have been reported, such as folie simultanée, in which two persons become psychotic
simultaneously and share the same delusion. Occasionally, more than two individuals are involved (e.g., folie á trois, quatre, cinq; also
folie á famille), but such cases are especially rare. The most common relationships in shared psychotic disorder are sister–sister,
husband–wife, and mother–child, but other combinations have also been described. Almost all cases involve members of a single family.

Correct Answer. c

(196). A 49-year-old male postman had undergone several operations to excise recurrent infections in both axillary lesions and perianal region.
The lesions are hydradenitis supperativa (Given Fig ).Which is true of these?

a. They arise from stratum corneum of skin.

b. They are noninflammatory conditions.

c. They always require surgical intervention.

d. They are usually caused by staphylococci and streptococci.

Solution. (d) They are usually caused by staphylococci and streptococci


Ref: Read the text below
Sol:
- Usually caused by staphylococci and streptococci.
- Hydradenitis supperativa is an infection of the apocrine glands and surrounding subcutaneous tissue and fascia, which most commonly
involves the axilla, groin, perineum, and perianal region.
- The periumbilical and areola region may be involved.
- In milder cases, local hygienic measures and tetracycline may be adequate; in more severe cases, wide excision is indicated.

Correct Answer. d

(197). An 18-year-old presents with a well circumscribed 2-cm mass in her right breast.The mass is painless and has a rubbery consistency and
discrete borders. It appears to move freely through the breast tissue. What is the likeliest diagnosis?

a. Carcinoma

b. Cyst

c. Fibroadenoma

d. Cystosarcoma phyllodes

Solution. (c) Fibroadenoma


Ref: Read the text below
Sol:
- Fibroadenomas are most often found in teenage girls.
- They are firm in consistency, clearly defined, and very mobile.
- The typical feature on palpation is that they appear to move freely through the breast tissue (“breast mouse”).

Copyright © 2014 Delhi Academy of Medical Sciences, All Rights Reserved. 76/116
Correct Answer. c

(198
Drug of choice for Imatinib resistant GIST is?
).

a. Lapatenib

b. Sorafenib

c. Sunitinib

d. Trastuzumab

Solution. (c) Sunitinib


Ref: Read the text below
Sol:

Correct Answer. c

(199). Right tri-segmentectomy includes removal of?

a. Segments II, VI, VII

b. Segments II, VII, VIII

c. Segments VI, VII, VIII

d. Segments V, VI, VII, VII, and IV (A and B)

Solution. (c) Segments VI, VII, VIII


Ref: Read the text below
Sol:

Correct Answer. c

Copyright © 2014 Delhi Academy of Medical Sciences, All Rights Reserved. 77/116
(200). Arterial infusions via the external carotid artery with methotrexate and 5-FU for head and neck carcinoma have shown a 50% response
rate. Widespread use, however, is limited. Why?

a. The internal carotid is inadvertently perfused in a large percentage of patients.

b. Ipsilateral facial slough has occurred in 3% of patients.

c. The response is transient, lasting only 2–3 months.

d. There is a prohibitive incidence of leukemia.

Solution. (c) The response is transient, lasting only 2–3 months.


Ref: Read the text below
Sol:
- The response is transient, lasting only 2–3 months.
- The results of this type of therapy, combined with those of artery occlusion, radiotherapy,or other modes of chemotherapy, require
further evaluation.

Correct Answer. c

(201). A 70 yrs male who presented with H/O of Dysphagia, Regurgitation and foul breath is best investigated by?

a. Barium Swallow.

b. Endosocpy.

c. Manometry.

d. EUS

Copyright © 2014 Delhi Academy of Medical Sciences, All Rights Reserved. 78/116
Solution. (a) Barium Swallow.
Ref: Read the text below
Sol:

Correct Answer. a

(202). Twenty-four hours after colon resection, urine output in a 70-year-old man is 10 mL/h. Blood chemistry analysis reveals sodium, 138
mEq/L; potassium, 6 mEq/L; chloride, 100 mEq/L; bicarbonate, 14 mEq/L. His metabolic abnormality is characterized by which of the
following?

a. Abdominal distension

b. Peaked T waves

c. Cardiac arrest in systole

d. J wave or Osborne wave

Copyright © 2014 Delhi Academy of Medical Sciences, All Rights Reserved. 79/116
Solution. (b) Peaked T waves
Ref: Read the text below
Sol:
- Hyperkalemia can manifest by GI or cardiovascular signs.
- GI symptoms include nausea, vomiting, intestinal colic, and diarrhea.
- Abdominal distension as a result of paralytic ileus is due to hypokalemia.
- An ECG is useful to monitor potassium levels.
- Hyperkalemia is characterized by peaked T waves. ECG changes also include ST-segment depression, widened QRS complex, and heart
block.
- Cardiac arrest occurs in diastole with increasing levels of potassium.
- Osborne (J) wave is seen in hypothermia.

Correct Answer. b

(203). MEN 2B is associated with an increased frequency with each of the following except.

a. Medullary cancer thyroid.

b. Mucosal neuroma.

c. Pheochromocytoma.

d. Hyperparathyroidism.

Solution. (d) Hyperparathyroidism.


Ref: Read the text below
Sol:

Correct Answer. d

(204). A 40 yr old 100 kg man is involved in a house fire with 45% burn (of total TBSA). He comes to the emergency 2 hrs since his injury. He
has not receive any resuscitation, after inserting 2 IV lines the rate of initial I.V. fluid should be.

a. 250 ml/ hr.

b. 500 ml/ hr.

c. 750 ml/ hr.

d. 1500 ml/ hr

Copyright © 2014 Delhi Academy of Medical Sciences, All Rights Reserved. 80/116
Solution. (c) 750 ml/ hr.
Ref: Read the text below
Sol:
- According to Parkland formula, IV fluid is calculated by the formula 4ml/ kg/ % of TBSA for the first 24 Hrs. with half given in first 8 hrs
and rest given in next 6 hrs.
- This formula predicts that this man will require 18000 ml of Isotonic fluid in first 24 hrs (4 × 100 × 45).
- One half should be given in first 8 hrs after injury.
- After a lapse of 2 hrs, 9000 ml fluid is required for next 6 hrs.
- Thus the initial rate should be 1500 ml/ Hr.

Correct Answer. c

(205). Spinnaker sign is seen in?

a. Pneumomediastinum.

b. Pneumoperitoneum

c. Pleural effusion.

d. Pneumothorax

Solution. (a) Pneumomediastinum.


Ref: Read the text below
Sol:

Correct Answer. a

(206). The abdominal compartment syndrome is characterized by increased abdominal pressure in all of the following symptoms except?

a. Increased peak inspiratory pressure.

b. Decreased urinary output.

c. Decreased venous return to heart.

d. Hypocarbia.

Copyright © 2014 Delhi Academy of Medical Sciences, All Rights Reserved. 81/116
Solution. (d) Hypocarbia.
Ref: Read the text below
Sol:
Abdominal compartment syndrome occurs predominantly in patients in profound shock, in patients requiring large amounts of resuscitation
fluids and blood, and in those with major visceral or vascular abdominal injuries.
Abdominal compartment syndrome is characterized by a sudden increase in intra-abdominal pressure, increased peak inspiratory pressure,
decreased urinary output, hypoxia, hypercapnia, and hypotension secondary to decreased venous return to the heart. The diagnosis is
confirmed by measuring bladder pressure, which ultimately represents intra- abdominal pressure. Treatment includes rapid decompression
of the elevated intra-abdominal pressure by opening the abdominal wound and performing a temporary closure of the abdominal wall by
Laparostomy.

Correct Answer. d

(207). A 60-year-old woman with mild hypertension is admitted for elective hysterectomy. On preoperative evaluation, she is found to have
osteoarthritis; over the previous 6 months, she had noted watery diarrhea that was becoming progressively worse. The serum potassium
is 3 mEq/L. Which is the most likely cause of hypokalemia?

a. Myoglobinemia

b. Villous adenoma of colon

c. High-output renal failure

d. Massive blood transfusion

Solution. (b) Villous adenoma of colon


Ref: Read the text below
Sol:
- Villous adenoma of colon can result in watery diarrhea and hypokalemia.
- Massive tissue injury producing myoglobinemia is associated with significant release of intracellular potassium.
- Massive blood transfusion results in release of large amounts of potassium.
- The ability to excrete potassium is impaired in high-output renal failure.
- Spironolactone is a potassium sparing diuretic.

Correct Answer. b

(208). A 14-year-old boy with a known bleeding tendency since infancy has severe epistaxis. Examination reveals an equinus contracture of the
right leg and a large hemarthrosis. Treatment should include which of the following?

a. Penicillamine

b. Transfusion of factor VIII to 30% of normal factor levels

c. Transfusion of factor VIII to 10% of normal factor levels

d. Platelet transfusion

Copyright © 2014 Delhi Academy of Medical Sciences, All Rights Reserved. 82/116
Solution. (b) Transfusion of factor VIII to 30% of normal factor levels
Ref: Read the text below
Sol:
- Spontaneous bleeding occurs when factor VIII is reduced below 2–3%.
- Once serious bleeding occurs, a higher factor VIII activity—probably approaching 30%—is required for adequate hemostasis.
- The half-life of factor VIII is 8–12 hours.
- In minor lesions, 10 U/kg body weight of factor VIII is administered. For severe lesions, the dosage is 40–50 U/kg body weight of factor
VIII.
- After major surgical procedures, factor VIII must be given daily for 7–10 days.
- Penicillamine is used to inhibit excess copper deposition (e.g., in Wilson’s disease).

Correct Answer. b

(209). The classic complete neck dissection for palpable adenopathy in the posterior triangle of the neck includes removal of which of the
following?

a. The transverse process, C2–C4

b. The spinal accessory nerve

c. Both thyroid lobes

d. The trapezius

Solution. (b) The spinal accessory nerve


Ref: Read the text below
Sol:
- The classic block dissection includes sternocleidomastoid muscle, the external and internal jugular veins, the spinal accessory nerve, the
submandibular gland, and the lymphatic tissue of the lateral compartment of the neck.
- Procedures that preserve muscle, nerve, or vessels are called modified neck dissection.

Correct Answer. b

(210). Best treatment for a 4 cm HCC in right lobe of liver with “Child’s Pugh score A” is?

a. Hepatic resection

b. Hepatic resection and Chemotherapy

c. Hepatic resection and radiotherapy

d. Hepatic resection and Liver transplant

Solution. (a) Hepatic resection


Ref: Read the text below
Sol:

Correct Answer. a

(211). Most common and most effective Bariatric surgery is?

a. Roux en Y gastric bypass

b. Laparoscopic Adjustable band gastroplasty

c. Sleeve gastrectomy

d. Duodenal switch

Copyright © 2014 Delhi Academy of Medical Sciences, All Rights Reserved. 83/116
Solution. (c) Sleeve gastrectomy
Ref: Read the text below
Sol:

Correct Answer. c

(212). Ischemia in intussusception is first seen at?

a. Neck

b. Apex

c. Inner wall

d. Middle wall

Copyright © 2014 Delhi Academy of Medical Sciences, All Rights Reserved. 84/116
Solution. (b) Apex
Ref: Read the text below
Sol:

Correct Answer. b

(213). Triangle of doom (In Laparoscopic hernia repair) is not bounded by?

a. Vas deference.

b. Testicular artery

c. Inferior epigastric artery

d. Peritoneal fold covering Psoas major.

Solution. (c) Inferior epigastric artery


Ref: Read the text below
Sol:
The Triangle of Doom
Is bounded by-
- Medially- vas deferens,
- Laterally- spermatic vessels,
- Inferiorly-peritoneal folds covering external iliac vessels and psoas.
This triangle contains external iliac artery and vessels, the deep circumflex iliac vein, the genital branch of genitofemoral nerve and
hidden by fascia the femoral nerve.

Correct Answer. c

Copyright © 2014 Delhi Academy of Medical Sciences, All Rights Reserved. 85/116
(214). A69-year-old man is informed that the cause of his dysphagis is a benign lesion. The barium swallow is shown in given fig. What should he
be told regarding benign tumors and cysts of the eosphagus?

a. They occur more commonly than malignant tumors.

b. They are symptomatic at an early age.

c. Diagnosis is best confirmed on chest x-ray.

d. Leiomyoma is the most common benign tumor encountered in the esophagus.

Solution. (d) Leiomyoma is the most common benign tumor encountered in the esophagus Ref: Read the text below
Sol:
- Leiomyoma is the most common benign tumor encountered in the esophagus. Malignant transformation is thought to be rare.
- Less than 10% of alimentary tract leiomyomas are found in the esophagus.
- They are composed of spindle cells and grow slowly and may progressivelycause obstructive symptoms.
- Leiomyomas are not referred to as a benign GIST.
- Other benign lesions are congenital or acquired cysts, adenomatous polyps, papillomas, lipomas, neurofibromas, and hemangiomas.

Correct Answer. d

(215). A32-year-old man undergoes a laparotomy for multiple organ injury resulting from trauma. He is discharged after 2 weeks in the hospital,
only to be readmitted 3 days later because of abdominal pain and sepsis. The CAT scan shows an accumulation of fluid in the subhepatic
space (Given Image). This space is likely to be directly related to an injury involving which structure?

a. Inferior pole of the right kidney

b. Stomach

c. Uncinate process of the pancreas

d. Aortic bifurcation

Solution. (b) Stomach


Ref: Read the text below
Sol:
- Subhepatic (intrahepatic) space infection usually occurs after surgery or peritonitis in the supracolic compartment.
- It is an unlikely amplication of biliary pancreatitis. Infections in the subhepatic space may extend to the infracolic compartment via the
paracolic gutter (of Morrison).
- In addition to the stomach, the subhepatic space may be involved with infection secondary to injury or diseases of the the gallbladder,
the first part of the duodenum, the anterior portions of the pancreas, or the liver.
- The uncinate lobe of the pancreas is the part of the head located posteriorly to the superior mesentric artery vein.

Correct Answer. b

Copyright © 2014 Delhi Academy of Medical Sciences, All Rights Reserved. 86/116
(216). A 47-year-old man presents with a 2.5-cm anal mass arising just proximal to the dentate line, involving > 1/2 of the circumference of the
anal canal, and fixed to the internal sphincter muscle. Biopsies show squamous cell carcinoma. Metastatic evaluation is negative. What
treatment would you recommend for this patient?

a. Low anterior resection with coloanal anastomosis

b. 5-Flourouricil, mitomycin C, and radiation therapy

c. APR with colostomy

d. Wide local excision with clear margins and close surveillance

Solution. b
This is a localized anal canal squamous cell car- cinoma. The primary treatment of localized squamous cell carci- noma of the anal canal
was defined by Norman Nigro, M.D. (the Nigro protocol), who demonstrated that combined chemoradia- tion therapy was much more
effective than radical surgery (APR), with high rates of anal preservation. The modified Nigro protocol in use today includes a course of
chemotherapy using 5-fluoro- uracil (5-FU) and mitomycin C and 6 weeks of radiation therapy to a dose of about 5400 cGy. Anal canal
carcinoma must be dif- ferentiated from anal margin carcinoma. Any squamous cell car- cinoma that has any part within the anal canal
(from the anal verge to the anorectal ring) is considered an anal canal carcinoma, while anal margin lesions are entirely outside of the
anal canal in the perineal skin. Anal margin carcinomas are treated by local excision with clear margins, which results in a high rate of
cure. APR is reserved for persistent or recurrent cancer or if the patient has previously received radiation therapy for other indications.
The modified Nigro protocol results in an initial complete response in 90% of patients. Salvage APR for persistence or recurrence results
in a cure rate of 25%–50%. So, the overall 5-year disease-free survival for anal canal carcinoma is about 80%.

Correct Answer. b

(217). Identify the abnormality shown in photograph

a. Hallux valgus

b. Hallux varus

c. Rheumatoid nodule

d. Subcutaneous nodule

Solution. (a)Hallux valgus


Ref: Read the text below
Sol:
HALLUX VALGUS
- Description: Lateral deviation of great toe to at metatarsophalangeal joint (Arrow in photograph)
- Causes: Rheumatoid arthritis, Idiopathic, high heel shoes
- Treatment: Mitchell osteotomy, mayo’s operation, keller’s operation

Correct Answer. a

(218). The most common injury following pelvic fracture is of:

a. Bladder.

b. Urethra.

c. Rectum.

d. Vagina.

Copyright © 2014 Delhi Academy of Medical Sciences, All Rights Reserved. 87/116
Solution. (b) Urethra.
Ref: Read below
Sol:
- Posterior urethral injuries are most commonly associated with pelvic fracture, with an incidence of 5%-10%. With an annual rate of 20
pelvic fractures per 100,000 population, these injuries are not uncommon.
- Anterior urethral injuries are less commonly diagnosed emergently; thus, the actual incidence is difficult to determine.
- However, many men with bulbar urethral strictures recall an antecedent perineal blunt injury or straddle injury, making the true
frequency of anterior urethral injury much higher.
- Penetrating injury to the urethra is rare, with major trauma centers reporting only a few per year

Correct Answer. b

(219). Paprika sign during debridement is crucial in management which of the following condition?

a. Chronic osteomyelitis

b. Osteosarcoma

c. Osteoid osteoma

d. Brodie’s abscess

Solution. (a) Chronic osteomyelitis


Ref: Read the text below
Sol:

Correct Answer. a

(220). Ponseti technique failure in children should be managed with –

a. Posteromedial soft tissue release

b. Anterolateral soft tissue release

c. Triple arthrodesis

d. Lateral closing wedge osteotomy of calcaneum

Solution. (a) Posteromedial soft tissue release


Ref: Read the text below
Sol:
- If treatment by manipulation and retention fails, operation should usually be undertaken before 9 months of age, mostly at 4-6 months.
- The idea behind early operation is to set the tarsal bones in normal relationship to one another and to remove deforming forces, thus
allowing the bones to develop in their normal shape from an early age.
- The procedure used is soft tissue release, i.e. postero-medial release (PMR). The taut structures on the postero-medial side are released.

Correct Answer. a

(221). Tenderness in anatomical snuff box is characteristic of which carpal bone fracture?

a. Scaphoid

Copyright © 2014 Delhi Academy of Medical Sciences, All Rights Reserved. 88/116
b. Capitate

c. Lunate

d. Triquetrum

Solution. (a) Scaphoid


Ref: Read the text below
Sol:
Clinical features of scaphoid fracture
- History of fall on outstretched hand
- Pain and fullness in the wrist, especially on radial side in the region of anatomical snuffbox.
- Movements at wrist are painful.
- Tenderness in the anatomical snuffbox
- A force transmitted along the axis of second metacarpal or pressure along the thumb produces pain in the region of scaphoid

Correct Answer. a

(222). Fracture shown in X-ray photographs is.

a. Colles fracture

b. Galeazzi fracture

c. Monteggia fracture

d. Smith fracture

Solution. (c) Monteggia fracture


Ref: Read the text below
Sol:
MONTEGGIA FRACTURE DISLOCATION (photograph)
- Fracture upper 1/3 of ulna
- Dislocation of head of radius

Correct Answer. c

Copyright © 2014 Delhi Academy of Medical Sciences, All Rights Reserved. 89/116
(223). The splint shown in Fig is used for?

a. Radial nerve palsy

b. Ulnar nerve palsy

c. Brachial plexus injury

d. Scoliosis

Solution. 223:(c) Brachial plexus injury


Ref: Read the text below
Sol:
- Image shows aeroplane splint used for brachial plexus injury

Correct Answer. c

(224). Osteomalacia predominantly affects the:

a. Spine.

b. Pelvis.

c. Skull bones.

d. Metatarsals.

Solution. (a) Spine.


Ref: Read below
Sol:
- Osteomalacia in adults starts insidiously as aches and pains in the lumbar (lower back) region and thighs, spreading later to the arms
and ribs.
- The pain is symmetrical, non-radiating and is accompanied by sensitivity in the involved bones. Proximal muscles are weak, and there is
difficulty in climbing up stairs and getting up from a squatting position.
- Due to demineralization bones become less rigid. Physical signs include deformities like triradiate pelvis and lordosis.
- The patient has a typical "waddling" gait. However, those physical signs may derive from a previous osteomalacial state, since bones do
not regain their original shape after they become deformed.

Correct Answer. a

(225). A patient comes to opd for pre operative assessment and on airway examination following points were noted:
Mouth opening > 3 finger breadths
Thyromental distance :6cms
Sternomental distance: 11 cms
How will this airway be categorised?

a. easy intubation

b. difficult intubation

c. difficult to comment

d. difficult bag and mask ventilation

Copyright © 2014 Delhi Academy of Medical Sciences, All Rights Reserved. 90/116
Solution. b
Assessment of difficult airway is done by “lemon”
L: look externally at signs of difficult airway (facial trauma high arched palate large tongue)
E: evaluate 3-3-2
Mouth opening > 3 fb
Hyomental distance> 3 fb
Thyroid to floor of mouth distance >2 fb
Thyromental distance > 6.5 cms
Sternomental distance > 12.5 cms (single most important predictor of difficult airway)
M: mallampatti grade ≥ 3
O: signs of obstruction (epiglottitis trauma)
N: restricted neck mobility

Correct Answer. b

(226). which of the following is a cardiostable agent except?

a. etomidate

b. nitrous oxide

c. ketamine

d. none

Solution. c. ketamine
Etomidate and nitrous oxide are cardiostable agents and have no effect on heart rate and blood pressure and so maintain cardiac output
as well.
Ketamine is unique because it has a indirect sympathomimetic action and so causes increase in systemic vascular resistance and thus
increase in heart rate and blood pressure but its direct action on the myocardium is causing depression.

Correct Answer. c

(227). which is safest inhaled anesthetic agent (environmentally)?

a. desflurane

b. sevoflurane

c. nitrous oxide

d. xenon

Solution. d xenon
Inhaled anesthetic agents have following effect on environment:
1: greenhouse effect global warming: maximum by desflurane
2: ozone depletion: max by halothane
3: overall impact of both of these effects is maximum by nitrous oxide (used in large quantities)
Xenon is a noble gas which is found natutally in the atmosphere and its inert and thus no environmental implications.

Correct Answer. d

(228). which if the following agents release histamine except?

a. cisatracurium

b. atracurium

c. mivacurium

d. d tubo curarane

Solution. d
Histamine releasing drug sin anesthesia are:
1) Atracurium
2) Mivacurium
3) Scoline
4) Dtc
5) Thiopentone
6) Morphine
7) Meperidine
All these agents have to be avoided in case of bronchial asthma as they can precipitate bronchospasm

Copyright © 2014 Delhi Academy of Medical Sciences, All Rights Reserved. 91/116
Correct Answer. d

(229). which of the following is true regarding neuromuscular monitoring except?

a. train of four is the most common used pattern

b. double burst stimulation shows better visual record of response than tof stimulation

c. patient who shows tof ration of 0.7 during recovery can be safely extubated

d. mechano myography is gold standard for recording of response.

Solution. c
Neuromuscular monitoring
1: the principle of neuromuscular monitoring is based on peripheral nerve stimulation and then recording of response.
2: most common pattern used is train of four in which 4 stimulations are given in 2 seconds
3: mechano myography is gold standard for recording of response
th st
4: during recovery we monitor tof ratio which is a ratio of intensity of 4 response to 1 response on tof stimulation
5: we administer reversal agent at tof ratio 0.7
6: patient can be safely extubated at tof ratio of 0.9

Correct Answer. c

(230). which process contributes to maximum loss of heat for a patient under anesthesia?

a. conduction

b. convection

c. sweating

d. radiation

Solution. d
Order of loss of heat for a patient under anesthesia is
Radiation> convection> conduction> evaporation

Correct Answer. d

(231). three days after knee arthroscopy under spinal anesthesia, a 55-year-old patient complains of double vision and difficulty hearing. The
other likely finding would be

a. Headache

b. Fever

c. Weakness in legs

d. Mental status changes

Solution. a
- Postdural puncture headaches (spinal headaches) usually develop within 12 to 72 hours after a dural puncture but may develop
immediately or take months to develop.
- The most characteristic symptom is a postural component in which the headache occurs in the upright position and is usually completely
gone when the patient is in the supine position.
- The headache is typically frontal and/or occipital in location.
- Other symptoms include nausea, vomiting, anorexia, visual disturbances (blurred vision, double vision, photophobia), and occasionally
hearing loss (routinely found with auditory testing)

Correct Answer. a

(232). during cpr how long should a pulse check last?

a. as long as it takes to find a pulse

b. no more than 2 seconds

c. no more than 5 seconds

d. no more than 10 seconds

Copyright © 2014 Delhi Academy of Medical Sciences, All Rights Reserved. 92/116
Solution. D: pulse check during cpr should be done after calling for help
Pulse and breathing should be assessed simultaneously
It should not be done for more than 10 seconds
There are different sites for different patients:
Adults: carotid artery
Child: femoral artery
Neonate: brachial artery

Correct Answer. d

(233). “Osteosclerotic” bone metastasis is found most commonly in which carcinoma?

a. Kidney

b. Thyroid

c. Lung

d. Prostate

Solution. (d) Prostate


Ref.: Read the text below
Sol:
- Pure osteosclerotic (osteoblastic) bone metastasis are seen in carcinoma prostate. Metastasis from prostate is commonly seen in the
spine.

Correct Answer. d

(234). TLD badges are worn for cumulative radiation measurement in radiation workers. TLD stands for –

a. Transient logging dosimeter

b. Tritium-Lutetium dosimeter

c. Thermoluminescent dosimeter

d. Total logging dosimeter

Solution. (c) Thermoluminescent dosimeter


Ref: Read the text below
Sol:
- TLD stands for thermoluminescent Dosimetry
-A thermoluminescent dosimeter, or TLD, is a type of radiationdosimeter. A TLD measures ionizing radiation exposure by measuring the
intensity of visible light emitted from a crystal in the detector when the crystal is heated.

Correct Answer. c

(235). A 21year old G1P0 present to OPD at 20weeks of gestation. The ultrasound image below shows the defect. Choose the most appropriate
diagnosis

a. Cystic hygroma

b. Anencephaly

c. Omphalocele

d. Encephalocele

Copyright © 2014 Delhi Academy of Medical Sciences, All Rights Reserved. 93/116
Solution. (b) Anencephaly
Ref: Read the text below
Sol:
ANENCEPHALY
- Characterized by absence of development of cranium with dystrophic brain tissue
- USG finding:
- Absent cranium with parts of occipital bone and mid brain present (photograph)
- No brain parenchymal tissue is seen in above orbits
- “frog eye” or “Mickey mouse” appearance: due to absent cranial bone/brain and bulging orbits
- Low crown rump length (CRL)
- May show evidence of polyhydramnios
Anencephaly diagnosis is suspected by 12-13 weeks and reliable around 15-16 weeks

Correct Answer. b

(236). Non visualized kidney on IVP is seen in:

a. Duplication

b. Polycystic kidney disease

c. Multicystic dysplastic kidney

d. Hypoplasia

Solution. (c) Multicystic dysplastic kidney


Ref: Read the text below
Sol:
- Multicystic dysplastic kidney is developmental disorder and doesn’t have any renal parenchyma and is non-visualized on IVP.

Correct Answer. c

(237). Study the given image and comment on Diagnosis:

a. Bochdalek hernia

b. Morgnani Hernia

c. Gastric volvulus

d. Hiatal hernia

Solution. (a) Bochdalek hernia


Ref: Read the text below
Sol:
- On conventional radiographs, the hernia may appear as a lung base soft-tissue opacity lesion seen posteriorly on lateral images.
- CT usually demonstrates fat above the diaphragm and is extremely beneficial in revealing organ entrapment.
- Coronal and sagittal reformatted images show the defect to best advantage.

Correct Answer. a

(238). Unilateral smooth enlarged kidney with striated nephrogram is seen in

a. Acute pyelonephritis

b. Chronic pyelopnephritis

Copyright © 2014 Delhi Academy of Medical Sciences, All Rights Reserved. 94/116
c. Autosomal recessive polycystic kidney disease

d. Multicystic dysplastic kidney

Solution. (a) Acute pyelonephritis


Ref: Read the text below
Sol:
- Striated nephrogram is seen in Autosomal recessive PCKD and in acute pyelonphritis.
- Other findings of Acute pyelopnephritis are on ultrasound: normal or enlarged kindey, with altered echogenicity areas showing areas of
reduced vascularity on Doppler.

Correct Answer. a

(239 Identify the most likely diagnosis on this CT abdomen.


).

a. Hepatic infarction

b. Ascites

c. Fatty Pancreas

d. Hepatocellular carcinoma

Solution. (a) Hepatic infarction


Ref: Read the text below
Sol:

Correct Answer. a

(240). True about ECHO modality all except:

a. 2D Echo is poor test for cardiac tamponade

b. 2D Echo is investigation of choice for pericardial effusion

Copyright © 2014 Delhi Academy of Medical Sciences, All Rights Reserved. 95/116
c. M-mode used for ventricular function

d. Dobutamine is used for conducting stress ECHO.

Solution. (a) 2D Echo is poor test for cardiac tamponade


Ref.: Read the text below
Sol:
- 2D ECHO is IOC for pericardial effusion, cardiac tamponade, vavular heart diseases and cardiomyopathy.
- In daily clinical practice, left Ventricular function is assessed by 2D-Echo and M-mode Echocardiography.
- Dobutamine is used during conduction of stress Echocardiography.
- M-mode Echocardiography: It is a 1D “ice pick” view through the heart. It is used to evaluate rapid motion of cardiac structures.

Correct Answer. a

(241). If mother received lithium treatment for bipolar disorder during pregnancy, the fetus is likely to show:

a. Neural tube defects

b. Facial defects

c. Urogenital defects

d. Cardiac defect

Solution. (d)Cardiac defect


Ref: Read the text below
Sol:
- Lithium is pregnancy category D drug. Lithium is known to cause various cardiac anomalies, especially Ebstein anomaly.
- There is an increased risk of this anomaly in babies exposed to lithium during the teratogenic period as compared with the normal
frequency of Ebstein anomaly which is about 1 in 20,000 births. Teratogenic effects of some drugs:
1. Phenytoin = Fetal hydantoin syndrome
2. Valproic acids = Spina bifida
3. Warfarin = Nasal hypoplasia, stippled vertebral and femoral epiphyses, agenesis of corpus callosum, Dandy-Walker malformation,
midline cerebellar atrophy, micro-ophthalmia, optic atrophy, blindness (Conradi's syndrome)
4. ACE inhibitors = Oligohydramnios, renal anomalies, neonatal renal failure, pulmonary hypoplasia, hypo calvaria, growth restriction,
death.

Correct Answer. d

(242). A 30-year-old primigravida at 39 weeks has been completely dilated and has been pushing for 3 h. She had taken epidural analgesia. She
is exhausted and has a temperature of 37.8°C.The fetal heart rate is 170/min with decreased variability. The patient's membranes have
been ruptured for over 24h. The patient's cervix is completely dilated and effaced, and the fetal head is visible at the introitus between
pushes. The fetal bones are at the +3 station. What is the most appropriate next step in the management of this patient?

a. Deliver the patient by cesarean section

b. Encourage the patient to continue to push after a short rest

c. Attempt operative delivery with forceps

d. Apply fundal pressure

Solution. (c)Attempt operative delivery with forceps


Ref: Read the text below
Ref: Read the text below
- Indications for an operative vaginal delivery with a vacuum extractor or forceps occur in situations where the fetal head is engaged, the
cervix is completely dilated, and there is a prolonged second stage, suspicion of potential fetal compromise, or need to shorten the second
stage for maternal benefit. In this situation, all the indications for operative delivery apply.
- This patient has, potential maternal and fetal compromise exists, since the patient has the clinical picture of chorioamnionitis and the
fetal heart rate is not reassuring.
- It is best to avoid cesarean section, since it would take more time to achieve and since the patient is infected.
- At full dilatation and a suitable station, forceps is faster than LSCS in baby delivery.

Correct Answer. c

(243). Treatment of cord prolapsed is based on all of the following factors, except:

a. Fetal viability

b. Fetal maturity

c. Fetal weight

Copyright © 2014 Delhi Academy of Medical Sciences, All Rights Reserved. 96/116
d. Cervical dilatation

Solution. (c)Fetal weight


Ref: Read the text below
Sol:
- Cord prolapsed is an obstetric emergency.
- After cord prolapse on exposure to external environment the whole cord goes into spasm, leading to severe decelerations in fetal heart
rate and fetal distress.
- If the fetus is alive and mature enough for survival, immediate delivery should be done.

Correct Answer. c

(244). Beyond which critical values shock index (heart rate/BP) in pregnancy is considered abnormal?

a. 0.9-1.1

b. 0.5-0.7

c. 0.3-0.5

d. 0.7–0.9

Solution. (a)0.9-1.1
Ref: Read the text below
Sol:
- Hypovolemic shock is the most common shock in obstetrics and obstetric hemorrhage (APH, PPH) is its MC cause.
- Hemorrhage is the most common cause of maternal mortality in developing countries like India.
- Shock Index = HR/Systolic BP
- Normal = 0.5-0.7
- Shock index >0.9 indicates state of shock that needs urgent resuscitation.

Correct Answer. a

(245). Use of oral contraceptives (OC) decreases the incidence of all of the following, except:

a. Ectopic pregnancy

b. Epithelial ovarian malignancy

c. Hepatic adenoma

d. Pelvic inflammatory disease

Solution. (c)Hepatic adenoma


Ref: Read the text below
Sol:
Non-contraceptive benefits/uses of OC pills

Correct Answer. c

(246). While evaluating a 30-year-old woman for infertility, you diagnose a bicornuate uterus. You explain that additional testing is necessary for
one organ system because of the woman's increased risk of congenital anomalies. Which is that organ system?

a. Skeletal

b. Tracheoesophageal

c. Urinary

d. Central nervous

Copyright © 2014 Delhi Academy of Medical Sciences, All Rights Reserved. 97/116
Solution. (c)Urinary
Ref: Read the text below
Sol:
- Failed fusion of the Mullerian ducts can give rise to several types of uterine anomalies, e.g. bicornuate uterus.
- This condition is associated with a higher risk of obstetric complications, such as an increase in the rate of second premature labor.
- Mal presentations such as breech and transverse lie are also more frequent. Also, prolonged labor (probably due to inadequate muscle
development in the uterus) and increased bleeding.
- An intravenous pyelogram or urinary tract ultrasound is mandatory in patients with Mullerian anomalies, since approximately 30% of
patients with Mullerian anomalies have coexisting congenital urinary tract anomalies.
- In bicornuate uterus (termed uterus bicornis unicollis), there is a double uterine cavity (bicornis) and a single cervix (unicollis) with a
normal vagina.

Correct Answer. c

(247). Best treatment modalities for dermoid cyst, diagnosed at 6 weeks of pregnancy is

a. LSCS along with removal of cyst

b. Only when it undergoes torsion

c. Immediately

d. At 14-16 weeks of pregnancy

Solution. (d) At 14-16 weeks of pregnancy


Ref: Read the text below
Sol:
Treatment of ovarian tumour in pregnancy
A) Uncomplicated- The best time of elective operation is between 14-18 weeks as the chances of abortion is less and access to the pedicle
is early
B) Beyond 36 weeks- The operation is batter to be with held till delivery and the tumour is removed as early in purpureum as possible
C) Complicated- The tumor should be removed irrespective of the period of gestation

Correct Answer. d

(248). Predisposing factors of carcinoma endometrium are all except

a. Obesity

b. Late menopause

c. Multiparity

d. Unopposed administration of estrogen

Solution. (c)Multiparity
Ref: Read the text below
Sol:
Multiparity is common in carcinoma cervix, while carcinoma of endometrium women are either nulliparous or of low parity
- Predisposing Factors in carcinoma Endometrium
- Peak incidence is 55-70 years
- An early menarche and late menopause is characteristic
- Unopposed and unsupervised administration of HRT
- Chronic non ovulatory cycles as seen in DUB
- Familial predisposing –familial Lynch II syndrome
- Tamoxifen given to women with breastcancer increases the risk of endometrial hyperplasia
- Combined oral hormonal pills

Correct Answer. c

(249). Following prostaglandins are used in induction of labor except

a. PGE1

b. PGE2

c. PG F2α

d. PGE2α

Copyright © 2014 Delhi Academy of Medical Sciences, All Rights Reserved. 98/116
Solution. C PGF2alpha
Ref: Read the text below
Sol:
- Both PGE2 and PGE2α have got and oxytocic effect on the pregnant uterus when used in appropriate dose.
- The probable mechanism of action is change in myometrial cell membrane permeability and /or alteration dose.
- The membrane bound Ca++,PGE2 is at leat 5 times more potent than PGE2α. PGE2α acts predominantly on the myometrium, while PGE2
acts mainly on the cervix.

Correct Answer. c

(250). when monitoring woman with partograph which statement is not true

a. If the graph crosses alert line you should put up oxytocin

b. The plotting begins at 4cm

c. Vaginal examination is done every 4 hrly

d. Contractions are monitored every 30mins

Solution. A
If the graph crosses alert line then referral to a higher center is done while intervention is done when plotting crosses action line. The
plotting begins when the patient enters active labour or 4 cm. per vaginal examinations are done every 4 hrly while FHR is measured
every 30mins and uterine contractions are also measured every 30 min interval.

Correct Answer. a

(251). Contraindications of this instrument include all of the following, except:

a. Face presentation

b. Prematurity

c. Maternal heart disease

d. IUFD

Solution. (c)Maternal heart disease


Ref: Read the text below
Sol:
Contraindications to vacuum extraction:
1. Operator inexperience
2. Inability to achieve a proper application
3. Uncertainty concerning fetal position/station
4. Suspicion of fetopelvic disproportion
5. High fetal head
6. Breech, face, brow presentations
7. Known or suspected fetal coagulation defect
8. Prematurity (fetuses <36 weeks' gestation)
9. Prior scalp sampling
10. IUFD (as chignon formation will not take place)
In heart disease patients, both forceps and vacuum can be applied to cut short second stage of labor.

Correct Answer. c

(252). Which surgical procedure has the highest incidence of ureteric injury?

a. Vaginal hysterectomy

b. Abdominal hysterectomy

c. Wertheim's hysterectomy

Copyright © 2014 Delhi Academy of Medical Sciences, All Rights Reserved. 99/116
d. Subtotal hysterectomy

Solution. (c)Wertheim's hysterectomy


Ref: Read the text below
Sol:
- Wertheim's hysterectomy requires dissection of the periureteral tissues and removing the lymphatics surrounding the course of the
ureter. This can devascularize the ureter causing ureteric injury/fistulas. The next most common cause of ureteric injury is abdominal
hysterectomy.
- Subtotal hysterectomy involves removal of only the body of uterus and keeping the cervix in situ. This can only be done if hysterectomy
is being done for benign conditions and not malignancy.
- It is done to prevent injury to bladder and ureter.

Correct Answer. c

(253). A 50-year-old woman is diagnosed with cervical cancer. Which lymph node group would be the first to be involved in metastatic spread of
this disease beyond the cervix and uterus?

a. Internal iliac nodes

b. Obturator nodes

c. External iliac nodes

d. Paracervical nodes

Solution. (d)Paracervical nodes


Ref: Read the text below
Sol:
- The main routes of spread of cervical cancer include vaginal mucosa, myometrium, paracervical lymphatics, and direct extension into
the parametrium.
- The prevalence of lymph node disease correlates with the stage of malignancy.
- Primary node groups involved in the spread of cervical cancer include the paracervical(sentinel node), parametrial, obturator,
hypogastric, external iliac, and sacral nodes, essentially in that order.
- Less commonly, there is involvement in the common iliac, inguinal, and para-aortic nodes.

Correct Answer. d

(254). A lady with abdominal mass was investigated. She was found to have bilateral ovarian masses with smooth surface. Study the given
microscopy and comment on the Diagnosis:

a. Dysgerminoma

b. Krukenberg tumor

c. Primary adenocarcinoma of the ovaries

d. Epithelial ovarian tumor

Copyright © 2014 Delhi Academy of Medical Sciences, All Rights Reserved. 100/116
Solution. (b)Krukenberg tumor
Ref: Read the text below
Sol:
Krukenberg tumor:
- Accounts for 30–40% of metastatic CAs to the ovary.
- Arises in the ovarian stroma and has characteristic mucin-filled signet ring cells.
- The primary is most frequently located in the stomach and less commonly in the colon, breast, or biliary tract. Rarely, thecervix and
bladder may be the primary site.
- They are usually bilateral and discovered when the primary is well advanced and hence survival is very poor.
- Treatment of primary carcinoma does not revert to Krukenberg tumor.
- The ovaries are enlarged and have a smooth surface.
- The shape of the ovary is maintained.
- There is no tendency of adhesion and the capsule remains intact.
- Cut surface shows waxy consistency with cystic spaces due to degeneration.

Correct Answer. b

(255). Sarcoma botryoides of vagina commonly occurs in which age group?

<10 years>a.

b. 20-30 years

c. 40–50 years

d. 50–60 years

Solution. (a)<10 years


Ref: Read the text below
Sol:
- Sarcoma botryoides or botryoid rhabdomyosarcoma is a subtype of embryonal rhabdomyosarcoma, which can be observed in the walls of
hollow, mucosa-lined structures such as the nasopharynx, common bile duct, urinary bladder of infants and young children or the vagina
in females, typically younger than age 8.
- The name comes from the gross appearance of “grape bunches” (botryoid in Greek).

Correct Answer. a

(256). In 40 days of menstrual cycle, the ovulation occurs at

a. 14th day
th
b. 20 day

c. 26th day

d. 30th day

Solution. (c)26th day


Ref: Read the text below
Sol:
- Ovulation in stimated to occur 14 days before the first day of the succeeding cycle and this interval is more or less fixed. In case of
irregular cycles, it is the follicular phase which varies but the luteal phase remains more or less constant at 14 days. So correct answer is
26 days
- Ovulation occurs approximately 10-12 hours after the LH peak or 32-36 hours after the onset of LH surge
- Ovulation coincides approximately 24-36 hours after the peak oestradiol level

Correct Answer. c

(257). All the following are the contraindications for the use of progesterone only pills except-

a. Pregnancy

b. Patient suffering from breast cancer

c. Peripheral vascular disease

d. Diabetes mellitus

Copyright © 2014 Delhi Academy of Medical Sciences, All Rights Reserved. 101/116
Solution. (d)Diabetes mellitus
Ref: Read the text below
Sol:
Contraindications for the use of progesterone only pills are as follows:
- Pregnancy - Unexplained vaginal bleeding
- Recent breast cancer - Arterial disease
- Thromboembolic disease

Correct Answer. d

(258). The foetal blood is separated from the syncytiotrophoblast with all except

a. Foetal blood capillary membrane

b. Mesenchyme inter villous blood space

c. Cytotrophoblast

d. Decidua-parietalis

Solution. (d)Decidua-parietalis
Ref: Read the text below
Sol:
Decidueparietalis-the rest of the deciduas lining the uterine cavity outside the site of implantation.
- Foetal blood now circulate through the villi, while meternal blood circulate through intervillous space
- Synocytiotrophoblast grows into the endometrium. As endometrium is eroded, some of its blood vessels are opened up, and blood from
them fills the lacunar spaces (intervillous space)
- Each trabeculus is initially made up entirely by syncytiotrophoblast. Now the cells of the cytotrophoblastbegain to multiple and grow
into each trabeculus. The trabeculus thus comes to have a central core of cytotrophoblast covered by an outer layer of syncytium. It is
surrounded by meternal blood filling the lacunar space. The trabeculuc is now called is primary villous.

Correct Answer. d

(259). In normal Puerperium, all are true regarding involution of uterus except

a. At the end of the 6 weeks, becomes non pregnant state

b. The number of muscle fibres is not decreased but there is substance al reduction of the myometrial cell size

c. At the end of the 4th week uterus becomes a pelvic organ

d. Weight of uterus about 80 gram at and of the 6 weeks

Solution. (c)At the end of the 4th week uterus becomes a pelvic organ
Ref: Read the text below
Sol:
INVOLUTION OF UTERUS
Following delivery the fundus lies above 13.5cm above the symphysis pubis. During the first 24 hours the
level remains constant, therefore there is steady decreased in height by 1.25 cm in 24 hours, so that by the
end of second week the uterus becomes a pelvic organ.

Correct Answer. c

(260). Most sensitive test of assessment of fetal well being is

a. Biparietal diameter

b. Arm circumference

c. Abdominal circumference

Copyright © 2014 Delhi Academy of Medical Sciences, All Rights Reserved. 102/116
d. Head circumference

Solution. C abdominal circumference


Ref: Read the text below
Sol:
Ultrasound parameters for gestational age assessment
First trimester- CRL
Second trimester-Biparietal diameter
Third trimester- Femur length
Best ultrasound marker for fetal growth is abdominal circumference

Correct Answer. c

(261). The best time to do chorionic villous sampling is:

a. 6-8 weeks

b. 8-9 weeks

c. 9-11 weeks

d. 11-13 weeks

Solution. (d)11-13 weeks


Ref: Read the text below
Sol:
- As per ACOG guidelines, chorionic villus biopsy should be done only after 10 weeks of gestation.
- This is done to avoid fetal risk of limb reduction defects and oromandibular defects and to ensure
retrieving adequate sample for processing. It can be done by abdominal route or vaginal route.
- Chorionic villus sampling below 10 weeks in criticized due to its adverse fetal effects.

Correct Answer. d

(262). Initial site of RBC production in fetus:

a. Gestational sac

b. Yolk sac

c. Placenta

d. Fetal bones

Solution. (b)Yolk sac


Ref: Read the text below
Sol:
- Embryonic and fetal hematopoiesis occurs in three phases: megaloblastic, hepatic, and myeloid.
- At each phase of RBC development, the sites of production and the cell composition change.
- Primitive erythropoiesis beginsin the yolk sac at 2-3 weeks postconception. By the end of the first trimester, the liver becomes the main
erythroid organ. Liver the primary source of red blood cells during the second trimester, and the bone marrow during the third trimester.

Correct Answer. b

Copyright © 2014 Delhi Academy of Medical Sciences, All Rights Reserved. 103/116
(263). Critical titer of hCG to visualize the gestational sac within the uterus on TVS is:

a. 1000-1500 micro IU/mL

b. 3500-6000 micro IU/mL

c. 500-800 micro IU/mL

d. 1500-2500 micro IU/mL

Solution. D
Ref: Read the text below
Sol:
Critical titer of hCG to visualize the gestational sac within the uterus:
TVS 2000 IU/L
TAS=6500 IU/mL

Correct Answer. d

(264). In modern day obstetrics, most common cause of ruptured uterus is:

a. Prolonged labor

b. Previous LSCS scar rupture

c. Forceps delivery

d. Internal podalic version

Solution. (b)Previous LSCS scar rupture


Ref: Read the text below
Sol:
- Disruption in the continuity of all uterine layers- endometrium, myometrium and serosa-anytime beyond 28 weeks of pregnancy is called
rupture of uterine.
- Its incidence varies from 1 in 2000 to 1 in 2000 deliveries. Prevalence of scar rupture is increased because of increase in lower segment
cesarean section (LSCS) rates.

Correct Answer. b

(265). Rate of cervical dilatation in active labour in primigravida is:

a. 0.8 cm/hour

b. 1.2 cm/hour

c. 105 cm/hour

d. 2 cm/hour

Solution. (b)1.2 cm/hour


Ref: Read the text below
Sol:
- Latent phase: This is the first 3cm of cervical dilatation, which is slow and takes about 8 hours in nulliparous and 4 hours in multiparous
women.
- Active phase: In this phase, normal rate of cervical dilatation is 1.2 cm/hour in primigravida and 1.5 cm/hour in multiparous women.

Correct Answer. b

(266). A 28 years old primigravida presents, at 18 weeks of gestational age, with right-sided groin pain. She describes the pain as sharp and
occurring with movement. She denies any change in urinary or bowel habits. She also denies any fever or chills. The application of a
heating pad helps to relieve the discomfort. The most likely etiology of this pain is:

a. Round ligament pain

b. Preterm labour

c. Kidney stone

d. Urinary tract infection

Copyright © 2014 Delhi Academy of Medical Sciences, All Rights Reserved. 104/116
Solution. (a)Round ligament pain
Ref: Read the text below
Sol:
- The patient is giving a classic description of round ligament pain. Each round ligament extends from the lateral portion of the uterus
below the tube and travels in a fold of peritoneum downward to the inguinal canal and insert in the upper portion of the labium majus.
This pain is greatly improved by avoiding sudden movements and by rising and sitting down gradually. Local heat and analgesics may also
help with pain control.
- The diagnosis of preterm labor is unlikely because the pain is localized to the groin area on one side and is alleviated with a heating pad.
- A urinary tract infection is unlikely because the patient has no urinary symptoms. Kidney stone is unlikely because, usually, the patients
would complain of pain the back, not low in the groin.

Correct Answer. a

(267). Anti phospholipids syndrome is characterized by all except

a. Associated with SLE

b. Thrombocytopenia & rash

c. Thrombotic disorders

d. Pancytopenia

Solution. (d)Pancytopenia
Ref: Read the text below
Sol:
ANTIPHOSPHOLIPID SYNDROME
- Develops in patients expressing antibodies to anionic phospholipids particularly β2 glycoprotein
- Half of the patients have no obvious cause, a few are pregnant, some are already receiving hemodialysis, or have a renal allograft, and
the rest have a primary glomerulonephritis (Nil lesion or membranous nephropathy) or rheumatologic disease such as SLE
- Cutaneous ulcers in APS

Correct Answer. d

(268). the following are true about gestational diabetes except

a. Start insulin when 1 hr PP is >=120mg/dl

b. Well controlled diabetes delivered at 39 weeks

c. Level 2 usg is recommended

d. Dose of folic acid recommended is 0.4mg

Solution. A
Insulin is started when woman with gestational diabetes on diabetic diet has a 2 hr PP value more than 120mg/dl
Level 2 usg is recommended for all pregnant women
Well controlled diabetes has to be delivered at and beyond 39 weeks and not earlier because of delayed lung maturation in a diabetic
pregnancy
Folic acid 0.4mg is to be given like all pregnant women starting 1 month before conception upto 3 months post conception.

Correct Answer. a

(269). latzko technique is used in

a. Incontinence of urine

b. Vesicovaginal fistula

c. Vesicouterine fistula

d. Urethrovaginal fistula

Solution. (b)Vesicovaginal fistula


Ref: Read the text below
Sol:
Vesicovaginal fistula
- Most vesicovaginal fistulae should be repaired vaginally.
- The Latzko procedure of denuding the vaginal epithelium all around the fistulous edge freshening the edge and approximating the wide
raw surface with rows of absorbable sutures is often successful.
- In case of extensive fibrosis, omental grafts, or gracilis muscle graft between the bladder and vaginal walls improves the blood supply at
the site of repair and promotes healing.

Copyright © 2014 Delhi Academy of Medical Sciences, All Rights Reserved. 105/116
Correct Answer. b

(270). the following are correct associations of OCPs except

a. Increase cervical cancer risk

b. Reduce ovarian cancer risk

c. Reduces endometrial cancer risk

d. Increases colon cancer risk

Solution. D
Women who have used oral contraceptives for 5 or more years have higher risk of cervical cancer and the risk is seen to decline over time
after woman stops using ocps
The endometrial and ovarian cancer risk is reduced with ocps
The risk of colon cancer is reduced by 15-20%

Correct Answer. d

(271). Maximum concentration of dextrose that can be given through peripheral vein is?

a. 5%

b. 10%

c. 12.5%

d. 25%

Solution. C
Explanation: The maximum concentration of dextrose that can be given through peripheral vein is 12.5%. Exceeding this concentration
can cause thrombophlebitis.

Correct Answer. c

(272). A 2 week old neonate was brought to the emergency room. The child was lethargic, had poor feeding & hypotonia. On examination, he is
hypotensive, tachycardiac, has bulging anterior fontanelle. Temp is 39.4o C & WBC count is found to be 16000 with 18% band cells, CRP
of 5 mg/dl.. What is the most likely diagnosis?

a. Klebsiella meningitis

b. E. coli sepsis

c. Group B streptococcal meningitis

d. Infantile hypotonia syndrome

Solution. C
Explanation:

Correct Answer. c

(273). A false positive APGAR score can be seen in all of the following conditions, except:

a. High fetal catecholamine levels

b. Prematurity

c. Bilateral choanal atresia

d. Spinal cord trauma

Copyright © 2014 Delhi Academy of Medical Sciences, All Rights Reserved. 106/116
Solution. A
Explanation:

Correct Answer. a

(274). Down syndrome children have decreased risk of developing the following as compared to general population?

a. Alzheimer disease

b. Neuroblastoma

c. Leukemia

d. Celiac disease

Solution. B
Explanation: Multiple recent studies have shown unexpected negative (protective) associations between Down syndrome and
comorbidities. Persons with Down syndrome have fewer-than-expected deaths caused by solid tumors and ischemic heart disease. There
was decreased risk of solid tumors in all age-groups with Down syndrome, including neuroblastomas and nephroblastomas in children and
epithelial tumors in adults. Ref. Nelson 21st Ed.

Correct Answer. b

(275). Central pontine myelinolysis can happen in children due to?

a. RL bolus in septic shock

b. Slow correction of hypernatremic dehydration

c. Under correction of hypernatremic dehydration

d. Rapid correction of hyponatremic dehydration

Solution. D
Explanation: As per Nelson 21st Ed, the initial goal in treating hyponatremia is correction of intravascular volume depletion with isotonic
fluid. An overly rapid (>12 mEq/L over 1st 24 hr) or overcorrection in the serum [Na +] (>135 mEq/L) is associated with an increased risk
of central pontine myelinolysis.

Correct Answer. d

Copyright © 2014 Delhi Academy of Medical Sciences, All Rights Reserved. 107/116
(276). Maximum transmission of which among the following is seen via breastmilk?

a. HIV

b. TB bacillus

c. CMV

d. EBV

Solution. C
Explanation: Transmission via breast milk: CMV>HIV>HBV>HCV

Correct Answer. c

(277). According to the latest RNTCP-2019 pediatric tuberculosis guidelines, indications to use corticosteroids in childhood TB include all
except?

a. TBM

b. TB uveitis

c. Tuberculomas

d. TB pericarditis

Solution. C
Explanation: Definite indications for concomitant steroid therapy include TBM, pericarditis, addison’s disease, miliary TB with alveolo-
capillary block and TB uveitis. The evidence in other forms of intracranial TB like tuberculomas is unclear. They may be used in
endobronchial tuberculosis, bronchial compression, mediastinal compression syndrome, pleurisy with severe distress, laryngeal TB, TB
IRIS and miliary disease with alveolo-capillary block. All children with TBM should be treated with adjuvant steroids regardless of the
disease severity. Dosage of prednisolone 1-2 mg/kg/day or dexamethasone 0.6 mg/kg/day or its equivalent is used for 4 weeks and then
tapered over next 4 weeks. Any steroid in equipotent doses can be used.

Correct Answer. c

(278). Incorrect statement regarding Wilms tumor is:

a. Hypertension may be the presenting feature in 5-15% patients

b. Invades major blood vessels

c. Bony metastasis is MC spread

d. Actinomycin D is highly effective

Solution. C
Explanation: Sites of metastasis in WT: Lungs (MC; may be present in some patients at the time of diagnosis), Regional LNs and Liver

Correct Answer. c

(279). True regarding Pallister-Killian syndrome is?

a. Shows genomic imprinting

b. Tetrasomy 12p

c. Hirsutism

d. Absence of digits in all cases

Solution. B
Explanation: Tetrasomy 12p
- Shows tetrasomy 12p 46, XX [12]/46, XX, +i(12p) [8] (mosaicism for an isochromosome 12p).
- Also known as Pallister-Killian syndrome.
- Features include sparse anterior scalp hair (more so temporal region), eyebrows, and eyelashes; prominent forehead; chubby cheeks;
long philtrum with thin upper lip and cupid-bow configuration; polydactyly; streaks of hyper- and hypopigmentation

Correct Answer. b

(280). Most common sequela associated with periventricular leukomalacia is?

Copyright © 2014 Delhi Academy of Medical Sciences, All Rights Reserved. 108/116
a. Spastic diplegia

b. Spastic quadriplegia

c. Seizures

d. Mental retardation

Solution. A
Explanation: Periventricular leukomalacia (PVL) is a pathologic hallmark of ischemic damage in preterm neonate. It commonly leads to
st
spastic diplegic type of cerebral palsy. As per Nelson 21 Ed, the most common neuropathologic finding in children with spastic diplegia
is PVL, which is visualized on MRI in more than 70% of cases. MRI typically shows scarring and shrinkage in the periventricular white
matter with compensatory enlargement of the cerebral ventricles.

Correct Answer. a

(281). Women with vitamin B12 deficiency presents with dysphagia and anemia. What is the syndrome mentioned in the presentation?

a. Plummer-Vinson syndrome

b. Eagle syndrome

c. Job's syndrome

d. Treacher Collin syndrome

Solution. (a)Plummer-Vinson syndrome


Ref: Read the text below
Sol:
- Triad of postcricoiddysphagia, oesophageal webs, and iron deficiency anemia constitutes Plummer Vinson syndrome. Due to iron
deficiency there occurs atrophy of the gastric mucosa leading to achlorhydria
- In PV syndrome due to atrophy of gastric mucosa therefore there is no IF available leading to poor absorption of vitamin B12.

Correct Answer. a

(282). Type 1 diabetes mellitus patients presents with nasal septal and palatal perforation with brownish black nasal discharge probable
diagnosis is:

a. Rhinosporidiosis

b. Aspergillus

c. Leprosy

d. Mucormycosis

Solution. (d)Mucormycosis
Ref: Read the text below
Sol:
- Diabetic patient (immunocompromised) with local tissue destruction with brownish black nasal discharge is suggestive of mucormycosis.
The black discharge due to the angio-invasive nature of mucormycosis (leading to vascular thrombosis and gangrene formation)
- Invasive aspergillosis and leprosy also present with nasal septal and palatal perforation but black discharge does not occur.
- Rhinosporidiosis presents with blood tinged nasal dischargefrom red polypoid mass of sporangia (mulberry or strawberry appearance)
and there is no local destruction.

Correct Answer. d

(283). A child presents with foul smelling ear discharge. On further exploration a small perforation is found in the pars flaccid of the tympanic
membrane. Most appropriate next step in the management would be:

a. Topical antibiotics and decongestants for 4 weeks

b. I/V antibiotics and follow up after a month

c. Tympanoplasty

d. Tympanomastoid exploration

Copyright © 2014 Delhi Academy of Medical Sciences, All Rights Reserved. 109/116
Solution. (d) Tympanomastoid exploration
Ref: Read the text below
Sol:
- Foul smelling discharge and perforation in the pars flaccid of the tympanic membrane indicates that it is an unsafe CSOM patient. So
the management is tympanomastoid exploration.
- Tympanoplasty is done as a part of Modified radical mastoidectomy in unsafe CSOM and per se is not the primary management. It is the
primary management of tubo-tympanic disease or safe CSOM.

Correct Answer. d

(284). Acute tonsillitis affects which nerve:

a. Glossopharyngeal nerve

b. Facial nerve

c. Trigeminal nerve

d. Vagus nerve

Solution. (a) Glossopharyngeal nerve


Ref: Read the text below
Sol:
Tonsillar branch of glossopharyngeal nerve glossopharyngeal nerve lies relation to posterior pole of tonsil and tonsillar bed. This leads to
earache in peritonsillar abscess and after tonsillectomy

Correct Answer. a

(285). Caldwell’s view is used mainly for:

a. Maxillary sinus

b. Frontal sinus

c. Ethmoidal sinus

d. Sphenoid sinus

Solution. (b) Frontal sinus


Ref: Read the text below
Sol:
- Caldwell’s view (or occipitofrontal view) is a radiography view of skull, where x-ray plate is angled at 20o to orbitomeatal line.
- The ray pass from behind the head and are perpendicular to radiographic plate.
- It is commonly used to get better view of frontal sinuses.

Correct Answer. b

(286). High tracheostomy indication is:

a. Vocal cord palsy

b. Laryngeal malignancy

c. Tracheomalacia

d. Foreign body obstructing airway

Solution. (b) Laryngeal malignancy


Ref: Read the text below
Sol:
TRACHEOTOMY
High Tracheotomy
Above the isthmus of thyroid gland against 2nd, 3rd and 4th tracheal rings
Done in malignancy of larynx presenting with strider where a laryngectomy has to be done later. This is because after laryngectomy, a
new tracheostoma has to be lower down. It can lead to perichondritis of cricoids cartilage and subglottic steno sis.
Mid Tracheotomy
At the level isthmus of thyroid gland against 2nd and 3rd tracheal rings
Done in case airway obstruction
1-Upper respiratory tract obstruction; Laryngeal, supralaryngeal and tracheal causes. (causes of stridor)
2-Lower respiratory tract obstruction: (secretary obstruction, Wet lung syndrome).

Correct Answer. b

Copyright © 2014 Delhi Academy of Medical Sciences, All Rights Reserved. 110/116
(287). A 2.5 years old fully immunised child developed sore throat, dysphagia, fever and stridor in quick succession. Within 2 hours of onset of
symptoms the child was very sick, pale and looked terrified. An hour later, the child became quiet and floppy. And within next one hour
the child expired. The most likely diagnosis was:

a. Acute laryngotracheal bronchitis

b. Acute epiglottitis

c. Laryngeal diphtheria

d. Foreign body aspiration

Solution. (b)Acute epiglottitis


Ref: Read the text below
Sol:
- Also since the child is immunized so practically no chances of diphtheria.
- Dysphagia and fast development of clinical symptoms point towards the acute epiglottitis.
- Even if the child is immunized against Hib, the other mentioned bacteria in the text can lead to epiglottitis.
- Laryngotracheobronchitis (croup): Dysphagia is not seen here.

Correct Answer. b

(288). A 45 year old male with history of nasal blockade for 6 months. On examination the patient had a nasal mass with mucin discharge. What
is the probable diagnosis based on the CT scan?

a. Antrochoanal polyp

b. Ethmoidal polyp

c. Maxillary carcinoma

d. Fungal sinusitis

Solution. (d)Fungal sinusitis


Ref: Read the text below
Sol:
- Computed tomography (CT) scan shows hyperattenuated focus with complete opacification of right maxillary sinus.
- The maxillary sinus is the most common involved followed by sphenoid, ethmoid and frontal

Correct Answer. d

Copyright © 2014 Delhi Academy of Medical Sciences, All Rights Reserved. 111/116
(289). A 35 year old male presented with nasal obstruction. His nasal endoscopy finding of one nostril is given. Which one is true regarding this
condition?

a. Best managed by intranasal polypectomy with snare

b. It bleeds on probing

c. Caldwell- luc operation in the treatment of choice in recurrent cases

d. It originates from maxillary sinus and goes to nasopharynx

Solution. (d)It originates from maxillary sinus and goes to nasopharynx


Ref: Read the text below
Sol:

Correct Answer. d

Copyright © 2014 Delhi Academy of Medical Sciences, All Rights Reserved. 112/116
(290). Corneal edema in hypoxic condition due to accumulation of

a. Lactate

b. Pyruvate

c. Glycogen

d. Carbon dioxide

Solution. A Lactate Ref: read the text below Stromal lactate accumulation can account for corneal edema osmotically following epithelial
hypoxia.

Correct Answer. a

(291). In ptosis operation:

a. brow suspension is the treatment of choice in severe congenital ptosis

b. Fasavella-Servant is the treatment of choice in patients without levator function

c. posterior approach is the recommended procedure in patient with previous ptosis surgery

d. maximal resection is recommended in patients with mitochondrial myopathy

Solution. A
In ptosis surgery:
o brow suspension allows the frontalis to perform upper lids elevation and is therefore recommended in severe congenital ptosis
o Fasavella-Servant is reserved for mild ptosis
o previous ptosis results in distortion of the normal lid anatomy making posterior approach unsuitable
o in mitochondrial myopathy poor Bell's phenomenon is often a feature and maximal resection can result in corneal exposure
o assessment of Bell's phenomenon and corneal sensation are essential prior to surgery to avoid exposure keratitis

Correct Answer. a

(292). Optic neuropathy have been reported with all except:

a. ethambutol

b. penicillin

c. chloramphenicol

d. isoniazid

Solution. B
Ref: Read the text below

Correct Answer. b

(293). The following are known features of ocular hypotony except:

a. optic oedema

b. cataract

Copyright © 2014 Delhi Academy of Medical Sciences, All Rights Reserved. 113/116
c. retinal detachment

d. corneal oedema

Solution. C
Ref: read the text below.
Hypotony:
occurs when the intraocular pressure is sufficient low to compromise the ocular function usually below 6 mmHg
may be caused by decreased aqueous production or excessive aqueous drainage
can results in:
- corneal oedema
- cataract formation
- uveal effusion
- macular oedema
- disc oedema
- choroidal folds
- choroidal detachment

Correct Answer. c

(294). Common ocular manifestation in Trisomy 13 is:

a. Capillary hemangioma

b. Bilateral microphthalmos

c. Neurofibroma

d. Dermoid cyst

Solution. b. Bilateral microphthalmos Explanation: The gross and microscopic eye findings in case of the 13-trisomy syndrome- Severe
microphthalmia, coloboma of the ciliary body, cataracts, detached retina, and retinal dysplasia

Correct Answer. b

(295). Following a RTA on fourth day a person developed proptosis and pain over right eye. On examination there is bruise on the eye and
forehead. What may be the diagnosis?

a. Fracture sphenoid

b. Cavernous sinus thrombosis

c. Internal carotid artery aneurysm

d. Carotid cavernous fistula

Solution. d. Carotid cavernous fistula Explanation: Carotid cavernous fistula: Abnormal communication between the cavernous sinus or
dural veins and the carotid arterial system Presentation days or weeks after head injury: Proptosis Conjunctival chemosis Whooshing
noise in head Ophthalmoplegia in 60-70 % due to ocular motor nerve damage The 6th nerve damage is most frequently affected because
of its free floating location within the cavernous sinus. Investigation: CT/ MRI Findings: Prominence of superior ophthalmic vein and
diffuse enlargement of extra-ocular muscles Choice of investigation→ Selective arterio-graphy of both internal and external carotid
arteries

Correct Answer. d

(296). Laser Trabeculoplasty is indicated in which of the following?

a. Neo-vascular glaucoma

b. Chronic angle closure glaucoma

c. Pseudo-exfoliative glaucoma

d. Uveitic glaucoma

Solution. c. Pseudo-exfoliative glaucoma


Explanation:
Indications of Laser Trabeculoplasty:
Pseudo-exfoliative glaucoma
Pigmentary glaucoma
Primary open angle glaucoma
Important point:
Laser Trabeculoplasty is performed with Argon or Nd-YAG laser.

Copyright © 2014 Delhi Academy of Medical Sciences, All Rights Reserved. 114/116
Correct Answer. c

(297). A patient presents with painless red eye with an IOP of 60 mmHg. What is the likely diagnosis?

a. Acute anterior uveitis

b. Glaucomatocyclitic crisis

c. Angle closure glaucoma

d. Chronic papillo-edema

Solution. b. Glaucomatocyclitic crisis


Explanation:
Posner Schlossman Syndrome (Glaucomatocyclitic crisis): Glaucomatocyclitic crisis is a condition with self-limited recurrent episodes of
markedly elevated intraocular pressure (IOP) with mild non-granulomatous anterior uveitis.

Correct Answer. b

(298). A distortion increases in both direction on wearing a lens. Which of the following is not true about this?

a. It is type of Aniseikonia

b. it is called as Pin Cushion effect

c. Seen on wearing a convex lenses

d. Cylindrical lenses increases the distortion

Solution. d. Cylindrical lenses increases the distortion


Explanation:
Pincushion distortion:
Magnification increases as distances from axis increases in the transverse plane.
Barrel distortion:
Magnification decreases as distances from axis increases.
Important point:
Pincushion distortion occurs in practically all spectacle corrected aphakes and is occasionally seen in high myopes after clear lens
extraction.

Correct Answer. d

(299). Which morphological type of cataract is most visually handicapping?

a. Cortical

b. Nuclear

c. Posterior sub-scapular

d. Zonular

Solution. c. Posterior sub-scapular


Explanation:
A posterior polar cataract is a round, discoid, opaque mass that is composed of malformed and distorted lens fibers located in the central
posterior sub-capsular area of the lens; right in the path of light on its way to the retina. A posterior sub-capsular cataract often interferes
with reading vision, reduces vision in bright light, and causes glare or halos around lights at night and maximum visual impairment as it
lies closer to the nodal point of eye.

Correct Answer. c

(300). Common ocular manifestation in Trisomy 13 is:

a. Capillary hemangioma

b. Bilateral microphthalmos

c. Neurofibroma

d. Dermoid cyst

Copyright © 2014 Delhi Academy of Medical Sciences, All Rights Reserved. 115/116
Solution. b. Bilateral microphthalmos
Explanation:
The gross and microscopic eye findings in case of the 13-trisomy syndrome- Severe microphthalmia, coloboma of the ciliary body,
cataracts, detached retina, and retinal dysplasia

Correct Answer. b
Test Answer
1.(b) 2.(c) 3.(a) 4.(b) 5.(b) 6.(b) 7.(a) 8.(b) 9.(a) 10.(d)

11.(c) 12.(d) 13.(a) 14.(b) 15.(a) 16.(b) 17.(b) 18.(b) 19.(b) 20.(a)

21.(b) 22.(b) 23.(a) 24.(c) 25.(d) 26.(b) 27.(b) 28.(c) 29.(a) 30.(b)

31.(b) 32.(a) 33.(c) 34.(c) 35.(a) 36.(d) 37.(d) 38.(a) 39.(a) 40.(a)

41.(d) 42.(b) 43.(b) 44.(d) 45.(b) 46.(d) 47.(b) 48.(b) 49.(a) 50.(c)

51.(c) 52.(c) 53.(b) 54.(d) 55.(c) 56.(a) 57.(c) 58.(a) 59.(b) 60.(c)

61.(d) 62.(c) 63.(d) 64.(d) 65.(d) 66.(d) 67.(c) 68.(c) 69.(d) 70.(c)

71.(c) 72.(a) 73.(a) 74.(c) 75.(c) 76.(b) 77.(c) 78.(d) 79.(d) 80.(a)

81.(a) 82.(d) 83.(c) 84.(c) 85.(b) 86.(d) 87.(c) 88.(d) 89.(c) 90.(d)

91.(c) 92.(a) 93.(d) 94.(d) 95.(c) 96.(c) 97.(a) 98.(c) 99.(d) 100.(b)

101.(d) 102.(a) 103.(b) 104.(d) 105.(a) 106.(a) 107.(b) 108.(d) 109.(d) 110.(d)

111.(a) 112.(d) 113.(a) 114.(b) 115.(b) 116.(c) 117.(a) 118.(d) 119.(c) 120.(d)

121.(b) 122.(b) 123.(b) 124.(c) 125.(c) 126.(c) 127.(b) 128.(b) 129.(d) 130.(c)

131.(a) 132.(b) 133.(a) 134.(a) 135.(c) 136.(a) 137.(c) 138.(c) 139.(a) 140.(a)

141.(d) 142.(c) 143.(b) 144.(d) 145.(a) 146.(c) 147.(c) 148.(d) 149.(b) 150.(d)

151.(c) 152.(d) 153.(d) 154.(d) 155.(d) 156.(a) 157.(d) 158.(a) 159.(d) 160.(c)

161.(c) 162.(b) 163.(a) 164.(b) 165.(a) 166.(c) 167.(a) 168.(c) 169.(d) 170.(d)

171.(a) 172.(c) 173.(a) 174.(d) 175.(c) 176.(c) 177.(a) 178.(c) 179.(b) 180.(c)

181.(c) 182.(a) 183.(d) 184.(b) 185.(b) 186.(d) 187.(c) 188.(c) 189.(c) 190.(b)

191.(b) 192.(a) 193.(d) 194.(c) 195.(c) 196.(d) 197.(c) 198.(c) 199.(c) 200.(c)

201.(a) 202.(b) 203.(d) 204.(c) 205.(a) 206.(d) 207.(b) 208.(b) 209.(b) 210.(a)

211.(c) 212.(b) 213.(c) 214.(d) 215.(b) 216.(b) 217.(a) 218.(b) 219.(a) 220.(a)

221.(a) 222.(c) 223.(c) 224.(a) 225.(b) 226.(c) 227.(d) 228.(d) 229.(c) 230.(d)

231.(a) 232.(d) 233.(d) 234.(c) 235.(b) 236.(c) 237.(a) 238.(a) 239.(a) 240.(a)

241.(d) 242.(c) 243.(c) 244.(a) 245.(c) 246.(c) 247.(d) 248.(c) 249.(c) 250.(a)

251.(c) 252.(c) 253.(d) 254.(b) 255.(a) 256.(c) 257.(d) 258.(d) 259.(c) 260.(c)

261.(d) 262.(b) 263.(d) 264.(b) 265.(b) 266.(a) 267.(d) 268.(a) 269.(b) 270.(d)

271.(c) 272.(c) 273.(a) 274.(b) 275.(d) 276.(c) 277.(c) 278.(c) 279.(b) 280.(a)

281.(a) 282.(d) 283.(d) 284.(a) 285.(b) 286.(b) 287.(b) 288.(d) 289.(d) 290.(a)

291.(a) 292.(b) 293.(c) 294.(b) 295.(d) 296.(c) 297.(b) 298.(d) 299.(c) 300.(b)

Copyright © 2014 Delhi Academy of Medical Sciences, All Rights Reserved. 116/116

Potrebbero piacerti anche